Is there any integral for the Golden Ratio?












134












$begingroup$


This is a curiosity. I was wondering about math important/famous constants, like $e$, $pi$, $gamma$ and obviously $phi$.



The first three ones are really well known, and there are lots of integrals and series whose results are simply those constants. For example:



$$ pi = 2 e intlimits_0^{+infty} frac{cos(x)}{x^2+1} text{d}x$$



$$ e = sum_{k = 0}^{+infty} frac{1}{k!}$$



$$ gamma = -intlimits_{-infty}^{+infty} x e^{x - e^{x}} text{d}x$$



My question is: is there some interesting integral $^*$ (or also some series) whose result is simply $phi$?



Note



$^*$ Interesting integral means that things like



$$intlimits_0^{+infty} e^{-frac{x}{phi}} text{d}x$$



are not a good answer to my question.










share|cite|improve this question











$endgroup$








  • 2




    $begingroup$
    You can skim this page, on WolframAlpha; e.g. Eq (12) and (13).
    $endgroup$
    – Clement C.
    Feb 14 '16 at 3:15








  • 2




    $begingroup$
    Related question introducing an infinite product for GR. And this question
    $endgroup$
    – Yuriy S
    Feb 14 '16 at 3:32








  • 6




    $begingroup$
    Also this. Somewhat famous locally :-)
    $endgroup$
    – Jyrki Lahtonen
    Feb 14 '16 at 9:45






  • 1




    $begingroup$
    In principle, any infinite sum can be expressed as an appropriate contour integral; thus, any of the known infinite sums for $phi$ can be expressed as contour integrals.
    $endgroup$
    – J. M. is not a mathematician
    Feb 15 '16 at 14:31










  • $begingroup$
    Hey guys could we get done proofs of these integrals please?
    $endgroup$
    – Faraz Masroor
    Feb 16 '16 at 12:44
















134












$begingroup$


This is a curiosity. I was wondering about math important/famous constants, like $e$, $pi$, $gamma$ and obviously $phi$.



The first three ones are really well known, and there are lots of integrals and series whose results are simply those constants. For example:



$$ pi = 2 e intlimits_0^{+infty} frac{cos(x)}{x^2+1} text{d}x$$



$$ e = sum_{k = 0}^{+infty} frac{1}{k!}$$



$$ gamma = -intlimits_{-infty}^{+infty} x e^{x - e^{x}} text{d}x$$



My question is: is there some interesting integral $^*$ (or also some series) whose result is simply $phi$?



Note



$^*$ Interesting integral means that things like



$$intlimits_0^{+infty} e^{-frac{x}{phi}} text{d}x$$



are not a good answer to my question.










share|cite|improve this question











$endgroup$








  • 2




    $begingroup$
    You can skim this page, on WolframAlpha; e.g. Eq (12) and (13).
    $endgroup$
    – Clement C.
    Feb 14 '16 at 3:15








  • 2




    $begingroup$
    Related question introducing an infinite product for GR. And this question
    $endgroup$
    – Yuriy S
    Feb 14 '16 at 3:32








  • 6




    $begingroup$
    Also this. Somewhat famous locally :-)
    $endgroup$
    – Jyrki Lahtonen
    Feb 14 '16 at 9:45






  • 1




    $begingroup$
    In principle, any infinite sum can be expressed as an appropriate contour integral; thus, any of the known infinite sums for $phi$ can be expressed as contour integrals.
    $endgroup$
    – J. M. is not a mathematician
    Feb 15 '16 at 14:31










  • $begingroup$
    Hey guys could we get done proofs of these integrals please?
    $endgroup$
    – Faraz Masroor
    Feb 16 '16 at 12:44














134












134








134


70



$begingroup$


This is a curiosity. I was wondering about math important/famous constants, like $e$, $pi$, $gamma$ and obviously $phi$.



The first three ones are really well known, and there are lots of integrals and series whose results are simply those constants. For example:



$$ pi = 2 e intlimits_0^{+infty} frac{cos(x)}{x^2+1} text{d}x$$



$$ e = sum_{k = 0}^{+infty} frac{1}{k!}$$



$$ gamma = -intlimits_{-infty}^{+infty} x e^{x - e^{x}} text{d}x$$



My question is: is there some interesting integral $^*$ (or also some series) whose result is simply $phi$?



Note



$^*$ Interesting integral means that things like



$$intlimits_0^{+infty} e^{-frac{x}{phi}} text{d}x$$



are not a good answer to my question.










share|cite|improve this question











$endgroup$




This is a curiosity. I was wondering about math important/famous constants, like $e$, $pi$, $gamma$ and obviously $phi$.



The first three ones are really well known, and there are lots of integrals and series whose results are simply those constants. For example:



$$ pi = 2 e intlimits_0^{+infty} frac{cos(x)}{x^2+1} text{d}x$$



$$ e = sum_{k = 0}^{+infty} frac{1}{k!}$$



$$ gamma = -intlimits_{-infty}^{+infty} x e^{x - e^{x}} text{d}x$$



My question is: is there some interesting integral $^*$ (or also some series) whose result is simply $phi$?



Note



$^*$ Interesting integral means that things like



$$intlimits_0^{+infty} e^{-frac{x}{phi}} text{d}x$$



are not a good answer to my question.







calculus integration big-list golden-ratio






share|cite|improve this question















share|cite|improve this question













share|cite|improve this question




share|cite|improve this question








edited May 5 '16 at 17:35









lisyarus

10.5k21433




10.5k21433










asked Feb 14 '16 at 2:57









Von NeumannVon Neumann

16.3k72545




16.3k72545








  • 2




    $begingroup$
    You can skim this page, on WolframAlpha; e.g. Eq (12) and (13).
    $endgroup$
    – Clement C.
    Feb 14 '16 at 3:15








  • 2




    $begingroup$
    Related question introducing an infinite product for GR. And this question
    $endgroup$
    – Yuriy S
    Feb 14 '16 at 3:32








  • 6




    $begingroup$
    Also this. Somewhat famous locally :-)
    $endgroup$
    – Jyrki Lahtonen
    Feb 14 '16 at 9:45






  • 1




    $begingroup$
    In principle, any infinite sum can be expressed as an appropriate contour integral; thus, any of the known infinite sums for $phi$ can be expressed as contour integrals.
    $endgroup$
    – J. M. is not a mathematician
    Feb 15 '16 at 14:31










  • $begingroup$
    Hey guys could we get done proofs of these integrals please?
    $endgroup$
    – Faraz Masroor
    Feb 16 '16 at 12:44














  • 2




    $begingroup$
    You can skim this page, on WolframAlpha; e.g. Eq (12) and (13).
    $endgroup$
    – Clement C.
    Feb 14 '16 at 3:15








  • 2




    $begingroup$
    Related question introducing an infinite product for GR. And this question
    $endgroup$
    – Yuriy S
    Feb 14 '16 at 3:32








  • 6




    $begingroup$
    Also this. Somewhat famous locally :-)
    $endgroup$
    – Jyrki Lahtonen
    Feb 14 '16 at 9:45






  • 1




    $begingroup$
    In principle, any infinite sum can be expressed as an appropriate contour integral; thus, any of the known infinite sums for $phi$ can be expressed as contour integrals.
    $endgroup$
    – J. M. is not a mathematician
    Feb 15 '16 at 14:31










  • $begingroup$
    Hey guys could we get done proofs of these integrals please?
    $endgroup$
    – Faraz Masroor
    Feb 16 '16 at 12:44








2




2




$begingroup$
You can skim this page, on WolframAlpha; e.g. Eq (12) and (13).
$endgroup$
– Clement C.
Feb 14 '16 at 3:15






$begingroup$
You can skim this page, on WolframAlpha; e.g. Eq (12) and (13).
$endgroup$
– Clement C.
Feb 14 '16 at 3:15






2




2




$begingroup$
Related question introducing an infinite product for GR. And this question
$endgroup$
– Yuriy S
Feb 14 '16 at 3:32






$begingroup$
Related question introducing an infinite product for GR. And this question
$endgroup$
– Yuriy S
Feb 14 '16 at 3:32






6




6




$begingroup$
Also this. Somewhat famous locally :-)
$endgroup$
– Jyrki Lahtonen
Feb 14 '16 at 9:45




$begingroup$
Also this. Somewhat famous locally :-)
$endgroup$
– Jyrki Lahtonen
Feb 14 '16 at 9:45




1




1




$begingroup$
In principle, any infinite sum can be expressed as an appropriate contour integral; thus, any of the known infinite sums for $phi$ can be expressed as contour integrals.
$endgroup$
– J. M. is not a mathematician
Feb 15 '16 at 14:31




$begingroup$
In principle, any infinite sum can be expressed as an appropriate contour integral; thus, any of the known infinite sums for $phi$ can be expressed as contour integrals.
$endgroup$
– J. M. is not a mathematician
Feb 15 '16 at 14:31












$begingroup$
Hey guys could we get done proofs of these integrals please?
$endgroup$
– Faraz Masroor
Feb 16 '16 at 12:44




$begingroup$
Hey guys could we get done proofs of these integrals please?
$endgroup$
– Faraz Masroor
Feb 16 '16 at 12:44










35 Answers
35






active

oldest

votes













1 2
next












31












$begingroup$

For $k>0$, we have



$$bbox[8pt,border:3px #FF69B4 solid]{color{red}{large int_0^infty ln left( frac{x^2-2kx+k^2}{x^2+2kxcos sqrt{pi^2-phi}+k^2}right) ;frac{mathrm dx}{x}=phi}}$$
I hope you find this integral interesting.



Extra:
$$bbox[8pt,border:3px #FF69B4 solid]{color{red}{large int_0^infty frac{x^{fracpi5-1}}{1+x^{2pi}} mathrm dx=phi}}$$






share|cite|improve this answer











$endgroup$













  • $begingroup$
    Click the box for the proof
    $endgroup$
    – Venus
    Jun 1 '16 at 8:41










  • $begingroup$
    this one is awesome ()+1
    $endgroup$
    – tired
    Aug 22 '16 at 8:23






  • 3




    $begingroup$
    I accepted your answer because you definitely wrote me what I wanted. Thank you so much for your beautiful elegant answer.
    $endgroup$
    – Von Neumann
    Sep 10 '16 at 10:28






  • 1




    $begingroup$
    It was a bit deflating to realize that, more generally, $$int_0^infty frac{x^{pi/k-1}}{1+x^{2pi}}mathrm dx=frac{1}{2}cscBig( frac{pi}{2k}Big)$$ and the second result was just the case $k=5$.
    $endgroup$
    – Tito Piezas III
    Nov 12 '16 at 16:15








  • 3




    $begingroup$
    @TitoPiezasIII Everything is a special case of something.
    $endgroup$
    – Simply Beautiful Art
    Jan 3 '17 at 13:41



















119












$begingroup$

Potentially interesting:



$$logvarphi=int_0^{1/2}frac{dx}{sqrt{x^2+1}}$$



Perhaps also worthy of consideration:



$$arctan frac{1}{varphi}=frac{int_0^2frac{1}{1+x^2}, dx}{int_0^2 dx}=frac{int_{-2}^2frac{1}{1+x^2}, dx}{int_{-2}^2 dx}$$



A development of the first integral:



$$logvarphi=frac{1}{2n-1}int_0^{frac{F_{2n}+F_{2n-2}}{2}}frac{dx}{sqrt{x^2+1}}$$



$$logvarphi=frac{1}{2n}int_1^{frac{F_{2n+1}+F_{2n-1}}{2}}frac{dx}{sqrt{x^2-1}}$$



which stem from the relationship $(x-varphi^m)(x-barvarphi^m)=x^2-(F_{m-1}+F_{m+1})x+(-1)^m$, where $barvarphi=frac{-1}{varphi}=1-varphi$ and $F_k$ is the $k$th Fibonacci number. I particularly enjoy:



$$logvarphi=frac{1}{3}int_0^{2}frac{dx}{sqrt{x^2+1}}$$
$$logvarphi=frac{1}{6}int_1^{9}frac{dx}{sqrt{x^2-1}}$$
$$logvarphi=frac{1}{9}int_0^{38}frac{dx}{sqrt{x^2+1}}$$
$$logvarphi=frac{1}{12}int_1^{161}frac{dx}{sqrt{x^2-1}}$$






share|cite|improve this answer











$endgroup$













  • $begingroup$
    Wow. Did you come up with this by yourself ?
    $endgroup$
    – user230452
    Feb 14 '16 at 4:25






  • 12




    $begingroup$
    @user230452 Unfortunately not! Stems from the fact that $text{arcsinh}{frac{1}{2}}=logvarphi$, and this connection comes by noting that $x^2-x-1=0implies frac{x-frac{1}{x}}{2}=frac{1}{2}$
    $endgroup$
    – πr8
    Feb 14 '16 at 4:28








  • 3




    $begingroup$
    What about $$int_0^{1/2}left(frac{x}{sqrt{x^2+1}}+3right),dx$$
    $endgroup$
    – Yves Daoust
    Feb 14 '16 at 18:02








  • 48




    $begingroup$
    +1 for the understatement, the neat answer and the awesome username. I assume you greet other $pi r8$s by saying "$Ar^k$" for some $kgeq2$.
    $endgroup$
    – David Richerby
    Feb 14 '16 at 18:23






  • 5




    $begingroup$
    @DavidRicherby Indeed - though I'm humbled enough by the reception this first integral seems to have received that I might be well-advised to go by $varphi$r$8$ from here onwards ^^.
    $endgroup$
    – πr8
    Feb 16 '16 at 14:00



















77












$begingroup$

In this answer, it is shown that
$$
int_0^inftyfrac{sqrt{x}}{x^2+2x+5}mathrm{d}x=fracpi{2sqrtphi}
$$






share|cite|improve this answer











$endgroup$









  • 4




    $begingroup$
    Awesome!! A strict link between $pi$ and $phi$, I love those things. Thank you!
    $endgroup$
    – Von Neumann
    Feb 14 '16 at 14:35










  • $begingroup$
    Brilliant!! Absolutely amazing
    $endgroup$
    – Albas
    Feb 14 '16 at 15:04










  • $begingroup$
    wow! this is incredible
    $endgroup$
    – Andres Mejia
    Feb 14 '16 at 16:45






  • 2




    $begingroup$
    So we know $pi=2eint_0^{infty}{cos(x)over x^2+1}text{d}x$ and $e=sum_{k=0}^{infty}{1over k!}$ from the OP, then this answer says $int_0^infty{sqrt{x}over x^2+2x+5}text{d}x={piover 2sqrt{Phi}}$. My immediate thought was to combine the above to get $Phi=left({sum_{k=0}^{infty}{1over k!}int_0^{infty}{cos(x)over x^2+1}text{d}x over int_0^infty{sqrt{x}over x^2+2x+5}text{d}x}right)^2$, which might be considered "interesting".
    $endgroup$
    – MichaelS
    Feb 14 '16 at 22:36










  • $begingroup$
    So very nice ! Somehow you perhaps can rope in $e$ too.
    $endgroup$
    – Narasimham
    Feb 15 '16 at 15:18



















48












$begingroup$

[EDIT:20190112 Another connection between $pi$ and $phi$ at the bottom] An identity derived from the Rogers-Ramanujan continued fraction ($R(q)$, not defined here) exhibits a $phi$ factor:



$$ frac{1}{(sqrt{phisqrt{5}})e^{2pi/5}} = 1+frac{e^{-2pi}}{1+frac{e^{-4pi}}{1+frac{e^{-6pi}}{1+frac{e^{-8pi}}{1+frac{e^{-10pi}}{1+frac{e^{-12pi}}{cdots}}}}}}$$



artist view of the identity



and one can then obtain a formula like:
$$ ln left( sqrt{4phi+3}-phi^2right) = -frac{1}{5}int_{e^{-2pi}}^1 frac{(1-t)^5(1-t^2)^5(1-t^3)^5 dots}{(1-t^5)(1-t^{10})(1-t^{15}) dots}frac{dt}{t}$$
which beautifully links integrals, $e$, $phi$ and $pi$. It is described for instance in Golden Ratio and a Ramanujan-Type Integral.



Not very practical though to obtain $phi$ rational approximations.



[EDIT] In M. D. Hirschhorn, A connection between $pi$ and $phi$, Fibonacci Quarterly, 2015, another asymptotic relation is:



$$ frac{1}{pi}=lim_{nto infty} 2n {5}^{1/4}sum_{k=0}^{n}binom{n}{k}^2binom{n+k}{k}/phi^{5n+5/2}$$






share|cite|improve this answer











$endgroup$









  • 15




    $begingroup$
    The genius of Ramanujan will always remain a mystery.. what a genius.
    $endgroup$
    – Von Neumann
    Feb 15 '16 at 14:33






  • 8




    $begingroup$
    And I believe it is a good thing that this remains a mystery.
    $endgroup$
    – Laurent Duval
    Feb 15 '16 at 14:58






  • 1




    $begingroup$
    mind... blown...
    $endgroup$
    – MichaelChirico
    Feb 18 '16 at 5:03










  • $begingroup$
    @FourierTransform right!
    $endgroup$
    – Fawad
    Oct 8 '16 at 9:14










  • $begingroup$
    The link seems dead, here is an archived version: pdf, html.
    $endgroup$
    – Vladimir Reshetnikov
    Jan 31 '18 at 3:08



















33












$begingroup$

$$int_{-1}^1 dx frac1x sqrt{frac{1+x}{1-x}} log{left (frac{2 x^2+2 x+1}{2 x^2-2 x+1}right )} = 4 pi operatorname{arccot}{sqrt{phi}}$$






share|cite|improve this answer











$endgroup$













  • $begingroup$
    There is a sign error in the log term
    $endgroup$
    – Cyclohexanol.
    Feb 14 '16 at 16:37










  • $begingroup$
    @LaplacianFourier: Thanks.
    $endgroup$
    – Ron Gordon
    Feb 14 '16 at 16:37






  • 4




    $begingroup$
    Ah yes, isn't this like the most upvoted post on this site? Always fun reading even though I don't know enough math do do it..
    $endgroup$
    – Faraz Masroor
    Feb 14 '16 at 21:48










  • $begingroup$
    @FarazMasroor: Actually, I think the 7th or 8th-most upvoted post. But thanks - if you want to learn feel free to ask questions!
    $endgroup$
    – Ron Gordon
    Feb 14 '16 at 21:51










  • $begingroup$
    ...might as well include a link: MSE 562964
    $endgroup$
    – Benjamin Dickman
    Feb 18 '16 at 7:27



















31












$begingroup$

Here's a series:



$$
phi = 1 + sum_{n=2}^infty frac{(-1)^{n}}{F_nF_{n-1}}
$$



where $F_n$ is the $n$th Fibonacci number.



To see this, rewrite the numerator using the identity $(-1)^n=F_{n+1}F_{n-1}-F_n^2$, at which point the summand becomes
$$
frac{F_{n+1}F_{n-1}-F_n^2}{F_nF_{n-1}}=frac{F_{n+1}}{F_n}-frac{F_n}{F_{n-1}}
$$
and so the sum telescopes: the partial sum ending at $n$ is equal to
$$
frac{F_{n+1}}{F_n}-frac{F_2}{F_1}=frac{F_{n+1}}{F_n} - 1
$$
which gives the original expression for the series via the limit $lim_{n to infty} frac{F_{n+1}}{F_n} = phi$.






share|cite|improve this answer









$endgroup$













  • $begingroup$
    Was this the first definition of golden ratio or did it have a definition before that ?
    $endgroup$
    – user230452
    Feb 14 '16 at 4:27












  • $begingroup$
    @user230452 $phi = frac { 1+ sqrt 5}2$
    $endgroup$
    – Ant
    Feb 14 '16 at 9:43










  • $begingroup$
    I mean, didn't that number come from the Fibonacci series itself or did it already have a definition and was found again in the Fibonacci series ?
    $endgroup$
    – user230452
    Feb 14 '16 at 10:23






  • 8




    $begingroup$
    @user230452 The golden ratio first arose as a ratio between quantities $a,b$ for which ratio $a:b$ is the same as ratio $a+b:a$. Hence, it came before (or at least independently of) Fibonacci numbers.
    $endgroup$
    – Wojowu
    Feb 14 '16 at 10:32






  • 3




    $begingroup$
    @Wojowu My point was just that the Fibonacci sequence is all about situations where you're dealing with $a$, $b$ and $a+b$, and so is the classical definition of the golden ratio. Whereas, for example, $(1+sqrt{5})/2$ is a completely different way of defining the same number. Anyway, I'm just nit-picking.
    $endgroup$
    – David Richerby
    Feb 14 '16 at 21:26



















30












$begingroup$

Based on the fact that $varphi = frac{1+sqrt{5}}{2}$:




$$varphi = int_4^5 frac32+frac1{4sqrt{x}} mathrm{d}x$$




Based on the fact that $varphi = 2cos(frac{pi}{5})$:




$$varphi = int_{tfrac{pi}{5}}^{tfrac{pi}{2}} 2sin(x) mathrm{d}x$$







share|cite|improve this answer









$endgroup$













  • $begingroup$
    I wanted to do that at first, but thought it wasn't 'interesting' by OP's standards
    $endgroup$
    – Yuriy S
    Feb 14 '16 at 12:31






  • 3




    $begingroup$
    @YuriyS I just took 'not interesting' as 'directly containing $varphi$, or a trivial variation on it'
    $endgroup$
    – wythagoras
    Feb 14 '16 at 12:35










  • $begingroup$
    Awesome, the second one is great!!
    $endgroup$
    – Von Neumann
    Feb 15 '16 at 14:34



















26












$begingroup$

$$int_0^{infty} frac{x^2}{1+x^{10}} , mathrm{d}x = frac{pi}{5 phi}.$$






share|cite|improve this answer









$endgroup$













  • $begingroup$
    Great! Another integral that relates two constants! Thank you!
    $endgroup$
    – Von Neumann
    Feb 14 '16 at 19:37










  • $begingroup$
    @KimPeek, there is an infinite number of integrals of this kind
    $endgroup$
    – Yuriy S
    Feb 14 '16 at 19:42






  • 4




    $begingroup$
    @YuriyS The more I see, the happier I am :D
    $endgroup$
    – Von Neumann
    Feb 14 '16 at 20:06






  • 5




    $begingroup$
    Although it adds nothing, I think having $5x^2$ on the left instead of $5$ at the denominator on the right looks even prettier (if possible)
    $endgroup$
    – Laurent Duval
    Feb 15 '16 at 7:14



















20












$begingroup$

$$int_0^{infty} frac{dx}{(1+x^phi)^phi}=1$$






share|cite|improve this answer











$endgroup$













  • $begingroup$
    Astounding beauty
    $endgroup$
    – Von Neumann
    Apr 24 '16 at 11:04






  • 1




    $begingroup$
    Another integral involving $phi$ that might be surprising at first sight :) $$int_0^inftyfrac1{1+x^2}frac1{1+x^phi}dx=fracpi4.$$
    $endgroup$
    – Vladimir Reshetnikov
    May 27 '16 at 19:27












  • $begingroup$
    @VladimirReshetnikov How would you solve both of your integrals one? My mind's blank, and I'm not able to simplify them adequately. :/
    $endgroup$
    – Kugelblitz
    Jun 17 '17 at 4:55










  • $begingroup$
    $displaystyleintfrac{dx}{(1+x^phi)^phi}=x , (1+x^phi)^{1-phi}color{gray}{+C}$, that can be checked by differentiation. The second one is really easy. Hint: if you replace $phi$ with $phi^2$, it will still have the same value.
    $endgroup$
    – Vladimir Reshetnikov
    Jun 17 '17 at 21:34





















19












$begingroup$

All the following is based on the simple fact that:



$$phi=2 cos left( frac{pi}{5} right)=2 sin left( frac{3pi}{10} right)$$



These integrals are the small sample of what we can build using this identity:



$$frac{1}{2 pi} int_0^{infty} frac{dx}{(1+x)x^{0.7}}=phi-1$$



$$frac{1}{1.4 pi} int_0^{infty} frac{dx}{(1+x)^2x^{0.7}}=phi-1$$



$$frac{1}{2 pi} int_0^{1} frac{dx}{(1-x)^{0.3}x^{0.7} }=phi-1$$



$$frac{5}{3 pi} int_0^{1} frac{x^{0.3}dx}{(1-x)^{0.3} }=phi-1$$



$$frac{1}{2 pi} int_1^{infty} frac{dx}{(x-1)^{0.3}x }=phi-1$$



$$frac{1}{0.21 pi} int_0^{infty} frac{x^{0.3}dx}{(1+x)^{3} }=phi-1$$



Take any tables of definite integrals, find any one that ends in a trig function and set the parameters to obtain $phi$.





You can find the following infinite product for $phi$ here



$$2 phi=prod_{k=0}^{infty}frac{100k(k+1)+5^2}{100k(k+1)+3^2}$$



It's converging slowly, see the link for the proof using the properties of Gamma function.



By numerical computation at $50000$ terms this infinite product gives only $5$ correct digits for $phi$, giving $1.618029$ instead of $1.618034$.



Using the infinite product for $cos(x)$, we get:



$$frac{phi}{2}=prod_{k=1}^{infty}left(1- frac{4}{5^2 (2k-1)^2} right)$$



This infinite product at $50000$ terms gives $phi=1.618035$, only $4$ correct digits. This is actually almost the same product, because if we rearrange it we get:



$$frac{phi}{2}=prod_{k=0}^{infty}left(frac{100 k (k+1)+21}{100 k (k+1)+25} right)$$



I suggest looking at this question for much more interesting product.






share|cite|improve this answer











$endgroup$





















    15












    $begingroup$

    The length of the logarithmic spiral $rho=e^{2theta}$ up to $theta=0$ is given by



    $$int_{-infty}^0sqrt{rho^2+dotrho^2}dtheta=int_{-infty}^0sqrt{1+2^2}e^{2theta}dtheta=phi-frac12.$$






    share|cite|improve this answer









    $endgroup$













    • $begingroup$
      Nice. Can you re-adjust the spiral so that length is $phi $ exactly ?
      $endgroup$
      – Narasimham
      Feb 15 '16 at 15:11












    • $begingroup$
      @Narasimham: I don't see an immediate way to achieve that.
      $endgroup$
      – Yves Daoust
      Feb 15 '16 at 15:22










    • $begingroup$
      You already have $sqrt{5}$ under your integral. Good example though
      $endgroup$
      – Yuriy S
      Feb 23 '16 at 23:16



















    11












    $begingroup$

    How about this one:




    $$int_0^1 frac{dx}{sqrt{x+sqrt{x+sqrt{x+cdots}}}}=frac{2}{phi}-ln phi$$




    There is an infinitely nested radical in the denominator.



    A finite one is also possible:




    $$int_0^{1/16} frac{dx}{sqrt{x+sqrt{x}}}=phi-2ln (phi)-frac12$$







    share|cite|improve this answer











    $endgroup$













    • $begingroup$
      The first one is AMAZING!! Thank you for having shared it! :O
      $endgroup$
      – Von Neumann
      Apr 11 '16 at 16:57










    • $begingroup$
      Might help in the second to note that $ln(phi+1)=2lnphi$
      $endgroup$
      – πr8
      May 3 '16 at 17:36






    • 1




      $begingroup$
      @TimeMaster The 1st integral is only a fancy representation of $$int_0^1 frac{2}{1+sqrt{1+4x}} dx$$
      $endgroup$
      – Sophie Agnesi
      Jun 10 '16 at 7:00










    • $begingroup$
      @SophieAgnesi, my secret is revealed! Curses!
      $endgroup$
      – Yuriy S
      Jun 10 '16 at 7:55



















    9












    $begingroup$

    $$int_0^infty x(2x-1),delta(x^2-x-1),dx$$





    Update:



    As pointed by Yuriy, we must take into account the derivative of the argument of the $delta$ function. This is why the corrective factor $2x-1$ appears.



    More generally,



    $$int_I x|g'(x)|delta(g(x)),dx$$ evaluates to the root of $g$ contained in the interval $I$, provided there is only one. The first factor $x$ can be replaced by any function $f(x)$ to yield the value of that function at the root.






    share|cite|improve this answer











    $endgroup$













    • $begingroup$
      Beautiful!! Dirac Delta. Very easy and elegant, thank you!
      $endgroup$
      – Von Neumann
      Feb 14 '16 at 17:43










    • $begingroup$
      A great idea, actually! We can do it for any algebraic number, it seems
      $endgroup$
      – Yuriy S
      Feb 14 '16 at 19:24










    • $begingroup$
      @YuriyS: yep, provided you isolate the desired root in an interval.
      $endgroup$
      – Yves Daoust
      Feb 14 '16 at 19:36










    • $begingroup$
      Actually, Wolframalpha gives another value for this integral: wolframalpha.com/input/…
      $endgroup$
      – Yuriy S
      Feb 14 '16 at 20:19










    • $begingroup$
      In general $delta [g(x)]=sum_k frac{delta (x-x_k)}{| g'(x_k)|}$
      $endgroup$
      – Yuriy S
      Feb 14 '16 at 20:25



















    7












    $begingroup$

    I am not taking credit for this. I am just posting this because it answers the question. I give Felix Marin and Olivier Oloa complete credit for these results.



    $$int_0^{pi/2} ln(1+4sin^2 x)text{ d}x=pilogleft(varphiright)$$



    and



    $$int_0^{pi/2} ln(1+4sin^4 x)text{ d}x=pilog frac{varphi+sqrt{varphi}}{2}$$



    Again, not mine. But they definitely deserve to be here






    share|cite|improve this answer









    $endgroup$













    • $begingroup$
      Beautiful! Thank you for having posted them here. The first one is so beautiful!!
      $endgroup$
      – Von Neumann
      Apr 2 '16 at 13:42



















    5












    $begingroup$

    $$
    int_0^1 frac{1+x^8}{1+x^{10}}dx=frac{pi}{phi^5-8}
    $$






    share|cite|improve this answer









    $endgroup$





















      5












      $begingroup$

      Consider the sequence



      $1,2,2,3,3,4,4,4,...$



      where $a_1=1,a_{n+1}in{a_n,a_n+1}$, and $a_n$ is the number of times $n$ occurs in the sequence. Then if we assume that $a_n$ grows asymptotically as $alpha n^beta$, we get



      $alpha=phi^{1/{phi^2}}$



      $beta=1/phi$.



      I saw this is a textbook problem on asymptotic analysis. It turns out that for all $n$ the asymptotic expression is well within one unit of the actual $a_n$.






      share|cite|improve this answer











      $endgroup$













      • $begingroup$
        I give up! How do I put braces around an explicitly written set!?!
        $endgroup$
        – Oscar Lanzi
        Apr 29 '16 at 10:46






      • 1




        $begingroup$
        Use { and } instead of the normal braces.
        $endgroup$
        – Marra
        May 2 '16 at 13:57



















      4












      $begingroup$

      So you said that series are OK, so I will offer a few:



      $$phi=frac{13}{8}+sum_{n=0}^infty frac{(-1)^{n+1}(2n+1)!}{n!(n+1)!4^{(2n+3)}}$$



      $$phi=2cos (pi/5)=2sum_{k=0}^infty frac{((-1)^k (pi/5)^{2 k}}{(2k)!}$$



      $$phi=frac{1}{2}+frac{sqrt{5}}{2}=frac{1}{2}+sum_{n=0}^infty 4^{-n}binom{1/2}{n}$$






      share|cite|improve this answer









      $endgroup$





















        4












        $begingroup$

        $$
        int_0^infty frac{1}{1+x^{10}}dx=frac{phipi}{5}
        $$






        share|cite|improve this answer









        $endgroup$













        • $begingroup$
          Awesome one!!!!
          $endgroup$
          – Von Neumann
          May 4 '16 at 9:27



















        3












        $begingroup$

        -I remember really liking this one:



        $$int_0^1 int_0^1 frac{text{dx dy}}{varphi^6-x^2y^2}=frac{pi^2-18log^2varphi}{24varphi^3}$$



        I most liked it because it was specific to $varphi$



        -Also, we can note this M.SE result (with some interpolation)



        $$int_0^1 frac{log (1+x^{alpha+sqrt{alpha^2-1}})}{1+x}text{dx}=$$$$frac{pi^2}{12}left(frac{alpha}{2}+sqrt{alpha^2-1}right)+log(varphi)log(2)log(sqrt{alpha+1}+sqrt{alpha-1})log(text{something})$$



        Perhaps someone can help me fill in $text{"something"}$






        share|cite|improve this answer











        $endgroup$





















          3












          $begingroup$

          Here is another one
          $$
          int_0^infty frac{1}{5^{frac{x}{4}}+5^{frac{1}{2}}-5^0}dx=phi
          $$






          share|cite|improve this answer









          $endgroup$





















            3












            $begingroup$

            This one is a bit messy.



            $$
            int_0^infty frac{1}{(sqrt5^x)^{2^{-(sqrt5-1)}}+sqrt5-1}dx=2^{phi^{-3}}cdotphi
            $$






            share|cite|improve this answer









            $endgroup$





















              3












              $begingroup$

              $$int_0^infty frac{1}{1+x^{frac{10}{3}}}dx=frac{3pi}{5phi}$$






              share|cite|improve this answer









              $endgroup$





















                3












                $begingroup$

                Let $ F_0=0, F_1=1 ; F_{n+1}=F_{n-1}+F_n
                $ be the Fibonacci numbers



                $zeta(s)$ is the zeta function. Then:



                $$
                prod_{n=1}^{infty}left[(-1)^{n+1}phi F_n+(-1)^nF_{n+1}right]^{n^{-(s+1)}}=phi^{-zeta(s)}
                $$






                share|cite|improve this answer











                $endgroup$













                • $begingroup$
                  This is Brilliant!!!
                  $endgroup$
                  – Von Neumann
                  May 4 '16 at 9:28



















                3












                $begingroup$

                Notice that $frac{2}{1+sqrt5}=frac{1}{phi}$



                $$int_0^1frac{2}{(1+sqrt5x)^2}dx=frac{1}{phi}$$






                share|cite|improve this answer









                $endgroup$





















                  3












                  $begingroup$

                  $$int_0^1 frac{200sqrt5(1-x^2)-300(1-x)^2}{ left[5sqrt5(1+x)^2-15(1-x^2)+2sqrt5(1-x)^2 right]^2}dx=(2phi+1)(phi+2)$$






                  share|cite|improve this answer









                  $endgroup$





















                    3












                    $begingroup$

                    Here is a collection of the series with reciprocal binomial coefficients.



                    $$sum_{n=0}^infty (-1)^n left( begin{matrix} 2n \ n end{matrix} right)^{-1}=frac{4}{5} left(1-frac{sqrt{5}}{5} ln phi right)$$



                    $$sum_{n=1}^infty frac{(-1)^n}{n} left( begin{matrix} 2n \ n end{matrix} right)^{-1}=-frac{2sqrt{5}}{5} ln phi$$



                    $$sum_{n=1}^infty frac{(-1)^n}{n^2} left( begin{matrix} 2n \ n end{matrix} right)^{-1}=-2 ln^2 phi$$



                    $$sum_{n=0}^infty frac{(-1)^n}{2n+1} left( begin{matrix} 2n \ n end{matrix} right)^{-1}=frac{4sqrt{5}}{5} ln phi$$



                    $$sum_{n=0}^infty frac{(-1)^n}{n+1} left( begin{matrix} 2n \ n end{matrix} right)^{-1}=frac{8sqrt{5}}{5} ln phi-4 ln^2 phi$$



                    $$sum_{n=2}^infty frac{(-1)^n}{n-1} left( begin{matrix} 2n \ n end{matrix} right)^{-1}=frac{3sqrt{5}}{5} ln phi-frac{1}{2}$$



                    $$sum_{n=2}^infty frac{(-1)^n}{(n-1)^2} left( begin{matrix} 2n \ n end{matrix} right)^{-1}=1-sqrt{5} ln phi+ ln^2 phi$$



                    $$sum_{n=2}^infty frac{(-1)^n}{n^2(n^2-1)} left( begin{matrix} 2n \ n end{matrix} right)^{-1}=4ln^2 phi-frac{sqrt{5}}{2} ln phi-frac{3}{8}$$



                    A one with $pi$:



                    $$sum_{n=0}^infty left( begin{matrix} 4n \ 2n end{matrix} right)^{-1}=frac{16}{15}+frac{sqrt{3}}{27} pi-frac{2sqrt{5}}{25} ln phi $$



                    Source here






                    share|cite|improve this answer











                    $endgroup$





















                      2












                      $begingroup$

                      Not exactly a series, but might also be of interest:



                      $$1-frac{1}{phi}=frac{1}{phi^2}=frac{1}{5} left(1+frac{1}{5} left(1+frac{1}{5} left(1+frac{1}{5} left(1+dots right)^2 right)^2 right)^2 right)^2$$





                      $$frac{1}{phi^4}=frac{1}{5} left(1-frac{1}{5} left(1-frac{1}{5} left(1-frac{1}{5} left(1-dots right)^2 right)^2 right)^2 right)^2$$



                      $$frac{1}{phi^4}=frac{1}{9} left(1+frac{1}{9} left(1+frac{1}{9} left(1+frac{1}{9} left(1+dots right)^2 right)^2 right)^2 right)^2$$






                      share|cite|improve this answer











                      $endgroup$





















                        2












                        $begingroup$

                        Here is another one




                        $$int_{-infty}^{+infty}e^{-x^2}cos (2x^2)mathrm dx=sqrt{phi piover 5}$$







                        share|cite|improve this answer









                        $endgroup$













                        • $begingroup$
                          Very cool one!!
                          $endgroup$
                          – Von Neumann
                          Apr 3 '17 at 7:54



















                        2












                        $begingroup$

                        We can prove the inequalities



                        $$frac{3}{2}<frac{8}{5}<phi<frac{13}{8}<frac{5}{3}$$



                        with representations



                        $$begin{align}phi&=frac{3}{2}+frac{1}{4}int_0^1 frac{dx}{sqrt{4+x}}\
                        \
                        phi&=frac{8}{5}+frac{1}{5}int_0^1 frac{dx}{sqrt{121+4x}}\
                        \
                        phi&=frac{13}{8}-frac{1}{16}int_0^1 frac{dx}{sqrt{80+x}}\
                        \
                        phi&=frac{5}{3}-frac{1}{3}int_0^1 frac{dx}{sqrt{45+4x}}\
                        end{align}$$






                        share|cite|improve this answer











                        $endgroup$





















                          2












                          $begingroup$

                          enter image description here



                          This is my attempt to find another integral representation for Golden ratio using some special function as shown in the above image !!!!!!!






                          share|cite|improve this answer









                          $endgroup$









                          • 2




                            $begingroup$
                            I already saw it in a past question of yours. It's really cool, can you prove it?
                            $endgroup$
                            – Von Neumann
                            Mar 22 '18 at 6:42






                          • 2




                            $begingroup$
                            Can we see all the proof please ?
                            $endgroup$
                            – Abr001am
                            May 8 '18 at 15:28















                          1 2
                          next











                          35 Answers
                          35






                          active

                          oldest

                          votes








                          35 Answers
                          35






                          active

                          oldest

                          votes









                          active

                          oldest

                          votes






                          active

                          oldest

                          votes








                          1 2
                          next










                          31












                          $begingroup$

                          For $k>0$, we have



                          $$bbox[8pt,border:3px #FF69B4 solid]{color{red}{large int_0^infty ln left( frac{x^2-2kx+k^2}{x^2+2kxcos sqrt{pi^2-phi}+k^2}right) ;frac{mathrm dx}{x}=phi}}$$
                          I hope you find this integral interesting.



                          Extra:
                          $$bbox[8pt,border:3px #FF69B4 solid]{color{red}{large int_0^infty frac{x^{fracpi5-1}}{1+x^{2pi}} mathrm dx=phi}}$$






                          share|cite|improve this answer











                          $endgroup$













                          • $begingroup$
                            Click the box for the proof
                            $endgroup$
                            – Venus
                            Jun 1 '16 at 8:41










                          • $begingroup$
                            this one is awesome ()+1
                            $endgroup$
                            – tired
                            Aug 22 '16 at 8:23






                          • 3




                            $begingroup$
                            I accepted your answer because you definitely wrote me what I wanted. Thank you so much for your beautiful elegant answer.
                            $endgroup$
                            – Von Neumann
                            Sep 10 '16 at 10:28






                          • 1




                            $begingroup$
                            It was a bit deflating to realize that, more generally, $$int_0^infty frac{x^{pi/k-1}}{1+x^{2pi}}mathrm dx=frac{1}{2}cscBig( frac{pi}{2k}Big)$$ and the second result was just the case $k=5$.
                            $endgroup$
                            – Tito Piezas III
                            Nov 12 '16 at 16:15








                          • 3




                            $begingroup$
                            @TitoPiezasIII Everything is a special case of something.
                            $endgroup$
                            – Simply Beautiful Art
                            Jan 3 '17 at 13:41
















                          31












                          $begingroup$

                          For $k>0$, we have



                          $$bbox[8pt,border:3px #FF69B4 solid]{color{red}{large int_0^infty ln left( frac{x^2-2kx+k^2}{x^2+2kxcos sqrt{pi^2-phi}+k^2}right) ;frac{mathrm dx}{x}=phi}}$$
                          I hope you find this integral interesting.



                          Extra:
                          $$bbox[8pt,border:3px #FF69B4 solid]{color{red}{large int_0^infty frac{x^{fracpi5-1}}{1+x^{2pi}} mathrm dx=phi}}$$






                          share|cite|improve this answer











                          $endgroup$













                          • $begingroup$
                            Click the box for the proof
                            $endgroup$
                            – Venus
                            Jun 1 '16 at 8:41










                          • $begingroup$
                            this one is awesome ()+1
                            $endgroup$
                            – tired
                            Aug 22 '16 at 8:23






                          • 3




                            $begingroup$
                            I accepted your answer because you definitely wrote me what I wanted. Thank you so much for your beautiful elegant answer.
                            $endgroup$
                            – Von Neumann
                            Sep 10 '16 at 10:28






                          • 1




                            $begingroup$
                            It was a bit deflating to realize that, more generally, $$int_0^infty frac{x^{pi/k-1}}{1+x^{2pi}}mathrm dx=frac{1}{2}cscBig( frac{pi}{2k}Big)$$ and the second result was just the case $k=5$.
                            $endgroup$
                            – Tito Piezas III
                            Nov 12 '16 at 16:15








                          • 3




                            $begingroup$
                            @TitoPiezasIII Everything is a special case of something.
                            $endgroup$
                            – Simply Beautiful Art
                            Jan 3 '17 at 13:41














                          31












                          31








                          31





                          $begingroup$

                          For $k>0$, we have



                          $$bbox[8pt,border:3px #FF69B4 solid]{color{red}{large int_0^infty ln left( frac{x^2-2kx+k^2}{x^2+2kxcos sqrt{pi^2-phi}+k^2}right) ;frac{mathrm dx}{x}=phi}}$$
                          I hope you find this integral interesting.



                          Extra:
                          $$bbox[8pt,border:3px #FF69B4 solid]{color{red}{large int_0^infty frac{x^{fracpi5-1}}{1+x^{2pi}} mathrm dx=phi}}$$






                          share|cite|improve this answer











                          $endgroup$



                          For $k>0$, we have



                          $$bbox[8pt,border:3px #FF69B4 solid]{color{red}{large int_0^infty ln left( frac{x^2-2kx+k^2}{x^2+2kxcos sqrt{pi^2-phi}+k^2}right) ;frac{mathrm dx}{x}=phi}}$$
                          I hope you find this integral interesting.



                          Extra:
                          $$bbox[8pt,border:3px #FF69B4 solid]{color{red}{large int_0^infty frac{x^{fracpi5-1}}{1+x^{2pi}} mathrm dx=phi}}$$







                          share|cite|improve this answer














                          share|cite|improve this answer



                          share|cite|improve this answer








                          edited Apr 13 '17 at 12:21









                          Community

                          1




                          1










                          answered Jun 1 '16 at 8:40









                          VenusVenus

                          6,89833686




                          6,89833686












                          • $begingroup$
                            Click the box for the proof
                            $endgroup$
                            – Venus
                            Jun 1 '16 at 8:41










                          • $begingroup$
                            this one is awesome ()+1
                            $endgroup$
                            – tired
                            Aug 22 '16 at 8:23






                          • 3




                            $begingroup$
                            I accepted your answer because you definitely wrote me what I wanted. Thank you so much for your beautiful elegant answer.
                            $endgroup$
                            – Von Neumann
                            Sep 10 '16 at 10:28






                          • 1




                            $begingroup$
                            It was a bit deflating to realize that, more generally, $$int_0^infty frac{x^{pi/k-1}}{1+x^{2pi}}mathrm dx=frac{1}{2}cscBig( frac{pi}{2k}Big)$$ and the second result was just the case $k=5$.
                            $endgroup$
                            – Tito Piezas III
                            Nov 12 '16 at 16:15








                          • 3




                            $begingroup$
                            @TitoPiezasIII Everything is a special case of something.
                            $endgroup$
                            – Simply Beautiful Art
                            Jan 3 '17 at 13:41


















                          • $begingroup$
                            Click the box for the proof
                            $endgroup$
                            – Venus
                            Jun 1 '16 at 8:41










                          • $begingroup$
                            this one is awesome ()+1
                            $endgroup$
                            – tired
                            Aug 22 '16 at 8:23






                          • 3




                            $begingroup$
                            I accepted your answer because you definitely wrote me what I wanted. Thank you so much for your beautiful elegant answer.
                            $endgroup$
                            – Von Neumann
                            Sep 10 '16 at 10:28






                          • 1




                            $begingroup$
                            It was a bit deflating to realize that, more generally, $$int_0^infty frac{x^{pi/k-1}}{1+x^{2pi}}mathrm dx=frac{1}{2}cscBig( frac{pi}{2k}Big)$$ and the second result was just the case $k=5$.
                            $endgroup$
                            – Tito Piezas III
                            Nov 12 '16 at 16:15








                          • 3




                            $begingroup$
                            @TitoPiezasIII Everything is a special case of something.
                            $endgroup$
                            – Simply Beautiful Art
                            Jan 3 '17 at 13:41
















                          $begingroup$
                          Click the box for the proof
                          $endgroup$
                          – Venus
                          Jun 1 '16 at 8:41




                          $begingroup$
                          Click the box for the proof
                          $endgroup$
                          – Venus
                          Jun 1 '16 at 8:41












                          $begingroup$
                          this one is awesome ()+1
                          $endgroup$
                          – tired
                          Aug 22 '16 at 8:23




                          $begingroup$
                          this one is awesome ()+1
                          $endgroup$
                          – tired
                          Aug 22 '16 at 8:23




                          3




                          3




                          $begingroup$
                          I accepted your answer because you definitely wrote me what I wanted. Thank you so much for your beautiful elegant answer.
                          $endgroup$
                          – Von Neumann
                          Sep 10 '16 at 10:28




                          $begingroup$
                          I accepted your answer because you definitely wrote me what I wanted. Thank you so much for your beautiful elegant answer.
                          $endgroup$
                          – Von Neumann
                          Sep 10 '16 at 10:28




                          1




                          1




                          $begingroup$
                          It was a bit deflating to realize that, more generally, $$int_0^infty frac{x^{pi/k-1}}{1+x^{2pi}}mathrm dx=frac{1}{2}cscBig( frac{pi}{2k}Big)$$ and the second result was just the case $k=5$.
                          $endgroup$
                          – Tito Piezas III
                          Nov 12 '16 at 16:15






                          $begingroup$
                          It was a bit deflating to realize that, more generally, $$int_0^infty frac{x^{pi/k-1}}{1+x^{2pi}}mathrm dx=frac{1}{2}cscBig( frac{pi}{2k}Big)$$ and the second result was just the case $k=5$.
                          $endgroup$
                          – Tito Piezas III
                          Nov 12 '16 at 16:15






                          3




                          3




                          $begingroup$
                          @TitoPiezasIII Everything is a special case of something.
                          $endgroup$
                          – Simply Beautiful Art
                          Jan 3 '17 at 13:41




                          $begingroup$
                          @TitoPiezasIII Everything is a special case of something.
                          $endgroup$
                          – Simply Beautiful Art
                          Jan 3 '17 at 13:41











                          119












                          $begingroup$

                          Potentially interesting:



                          $$logvarphi=int_0^{1/2}frac{dx}{sqrt{x^2+1}}$$



                          Perhaps also worthy of consideration:



                          $$arctan frac{1}{varphi}=frac{int_0^2frac{1}{1+x^2}, dx}{int_0^2 dx}=frac{int_{-2}^2frac{1}{1+x^2}, dx}{int_{-2}^2 dx}$$



                          A development of the first integral:



                          $$logvarphi=frac{1}{2n-1}int_0^{frac{F_{2n}+F_{2n-2}}{2}}frac{dx}{sqrt{x^2+1}}$$



                          $$logvarphi=frac{1}{2n}int_1^{frac{F_{2n+1}+F_{2n-1}}{2}}frac{dx}{sqrt{x^2-1}}$$



                          which stem from the relationship $(x-varphi^m)(x-barvarphi^m)=x^2-(F_{m-1}+F_{m+1})x+(-1)^m$, where $barvarphi=frac{-1}{varphi}=1-varphi$ and $F_k$ is the $k$th Fibonacci number. I particularly enjoy:



                          $$logvarphi=frac{1}{3}int_0^{2}frac{dx}{sqrt{x^2+1}}$$
                          $$logvarphi=frac{1}{6}int_1^{9}frac{dx}{sqrt{x^2-1}}$$
                          $$logvarphi=frac{1}{9}int_0^{38}frac{dx}{sqrt{x^2+1}}$$
                          $$logvarphi=frac{1}{12}int_1^{161}frac{dx}{sqrt{x^2-1}}$$






                          share|cite|improve this answer











                          $endgroup$













                          • $begingroup$
                            Wow. Did you come up with this by yourself ?
                            $endgroup$
                            – user230452
                            Feb 14 '16 at 4:25






                          • 12




                            $begingroup$
                            @user230452 Unfortunately not! Stems from the fact that $text{arcsinh}{frac{1}{2}}=logvarphi$, and this connection comes by noting that $x^2-x-1=0implies frac{x-frac{1}{x}}{2}=frac{1}{2}$
                            $endgroup$
                            – πr8
                            Feb 14 '16 at 4:28








                          • 3




                            $begingroup$
                            What about $$int_0^{1/2}left(frac{x}{sqrt{x^2+1}}+3right),dx$$
                            $endgroup$
                            – Yves Daoust
                            Feb 14 '16 at 18:02








                          • 48




                            $begingroup$
                            +1 for the understatement, the neat answer and the awesome username. I assume you greet other $pi r8$s by saying "$Ar^k$" for some $kgeq2$.
                            $endgroup$
                            – David Richerby
                            Feb 14 '16 at 18:23






                          • 5




                            $begingroup$
                            @DavidRicherby Indeed - though I'm humbled enough by the reception this first integral seems to have received that I might be well-advised to go by $varphi$r$8$ from here onwards ^^.
                            $endgroup$
                            – πr8
                            Feb 16 '16 at 14:00
















                          119












                          $begingroup$

                          Potentially interesting:



                          $$logvarphi=int_0^{1/2}frac{dx}{sqrt{x^2+1}}$$



                          Perhaps also worthy of consideration:



                          $$arctan frac{1}{varphi}=frac{int_0^2frac{1}{1+x^2}, dx}{int_0^2 dx}=frac{int_{-2}^2frac{1}{1+x^2}, dx}{int_{-2}^2 dx}$$



                          A development of the first integral:



                          $$logvarphi=frac{1}{2n-1}int_0^{frac{F_{2n}+F_{2n-2}}{2}}frac{dx}{sqrt{x^2+1}}$$



                          $$logvarphi=frac{1}{2n}int_1^{frac{F_{2n+1}+F_{2n-1}}{2}}frac{dx}{sqrt{x^2-1}}$$



                          which stem from the relationship $(x-varphi^m)(x-barvarphi^m)=x^2-(F_{m-1}+F_{m+1})x+(-1)^m$, where $barvarphi=frac{-1}{varphi}=1-varphi$ and $F_k$ is the $k$th Fibonacci number. I particularly enjoy:



                          $$logvarphi=frac{1}{3}int_0^{2}frac{dx}{sqrt{x^2+1}}$$
                          $$logvarphi=frac{1}{6}int_1^{9}frac{dx}{sqrt{x^2-1}}$$
                          $$logvarphi=frac{1}{9}int_0^{38}frac{dx}{sqrt{x^2+1}}$$
                          $$logvarphi=frac{1}{12}int_1^{161}frac{dx}{sqrt{x^2-1}}$$






                          share|cite|improve this answer











                          $endgroup$













                          • $begingroup$
                            Wow. Did you come up with this by yourself ?
                            $endgroup$
                            – user230452
                            Feb 14 '16 at 4:25






                          • 12




                            $begingroup$
                            @user230452 Unfortunately not! Stems from the fact that $text{arcsinh}{frac{1}{2}}=logvarphi$, and this connection comes by noting that $x^2-x-1=0implies frac{x-frac{1}{x}}{2}=frac{1}{2}$
                            $endgroup$
                            – πr8
                            Feb 14 '16 at 4:28








                          • 3




                            $begingroup$
                            What about $$int_0^{1/2}left(frac{x}{sqrt{x^2+1}}+3right),dx$$
                            $endgroup$
                            – Yves Daoust
                            Feb 14 '16 at 18:02








                          • 48




                            $begingroup$
                            +1 for the understatement, the neat answer and the awesome username. I assume you greet other $pi r8$s by saying "$Ar^k$" for some $kgeq2$.
                            $endgroup$
                            – David Richerby
                            Feb 14 '16 at 18:23






                          • 5




                            $begingroup$
                            @DavidRicherby Indeed - though I'm humbled enough by the reception this first integral seems to have received that I might be well-advised to go by $varphi$r$8$ from here onwards ^^.
                            $endgroup$
                            – πr8
                            Feb 16 '16 at 14:00














                          119












                          119








                          119





                          $begingroup$

                          Potentially interesting:



                          $$logvarphi=int_0^{1/2}frac{dx}{sqrt{x^2+1}}$$



                          Perhaps also worthy of consideration:



                          $$arctan frac{1}{varphi}=frac{int_0^2frac{1}{1+x^2}, dx}{int_0^2 dx}=frac{int_{-2}^2frac{1}{1+x^2}, dx}{int_{-2}^2 dx}$$



                          A development of the first integral:



                          $$logvarphi=frac{1}{2n-1}int_0^{frac{F_{2n}+F_{2n-2}}{2}}frac{dx}{sqrt{x^2+1}}$$



                          $$logvarphi=frac{1}{2n}int_1^{frac{F_{2n+1}+F_{2n-1}}{2}}frac{dx}{sqrt{x^2-1}}$$



                          which stem from the relationship $(x-varphi^m)(x-barvarphi^m)=x^2-(F_{m-1}+F_{m+1})x+(-1)^m$, where $barvarphi=frac{-1}{varphi}=1-varphi$ and $F_k$ is the $k$th Fibonacci number. I particularly enjoy:



                          $$logvarphi=frac{1}{3}int_0^{2}frac{dx}{sqrt{x^2+1}}$$
                          $$logvarphi=frac{1}{6}int_1^{9}frac{dx}{sqrt{x^2-1}}$$
                          $$logvarphi=frac{1}{9}int_0^{38}frac{dx}{sqrt{x^2+1}}$$
                          $$logvarphi=frac{1}{12}int_1^{161}frac{dx}{sqrt{x^2-1}}$$






                          share|cite|improve this answer











                          $endgroup$



                          Potentially interesting:



                          $$logvarphi=int_0^{1/2}frac{dx}{sqrt{x^2+1}}$$



                          Perhaps also worthy of consideration:



                          $$arctan frac{1}{varphi}=frac{int_0^2frac{1}{1+x^2}, dx}{int_0^2 dx}=frac{int_{-2}^2frac{1}{1+x^2}, dx}{int_{-2}^2 dx}$$



                          A development of the first integral:



                          $$logvarphi=frac{1}{2n-1}int_0^{frac{F_{2n}+F_{2n-2}}{2}}frac{dx}{sqrt{x^2+1}}$$



                          $$logvarphi=frac{1}{2n}int_1^{frac{F_{2n+1}+F_{2n-1}}{2}}frac{dx}{sqrt{x^2-1}}$$



                          which stem from the relationship $(x-varphi^m)(x-barvarphi^m)=x^2-(F_{m-1}+F_{m+1})x+(-1)^m$, where $barvarphi=frac{-1}{varphi}=1-varphi$ and $F_k$ is the $k$th Fibonacci number. I particularly enjoy:



                          $$logvarphi=frac{1}{3}int_0^{2}frac{dx}{sqrt{x^2+1}}$$
                          $$logvarphi=frac{1}{6}int_1^{9}frac{dx}{sqrt{x^2-1}}$$
                          $$logvarphi=frac{1}{9}int_0^{38}frac{dx}{sqrt{x^2+1}}$$
                          $$logvarphi=frac{1}{12}int_1^{161}frac{dx}{sqrt{x^2-1}}$$







                          share|cite|improve this answer














                          share|cite|improve this answer



                          share|cite|improve this answer








                          edited Feb 16 '16 at 13:57

























                          answered Feb 14 '16 at 3:15









                          πr8πr8

                          9,84831024




                          9,84831024












                          • $begingroup$
                            Wow. Did you come up with this by yourself ?
                            $endgroup$
                            – user230452
                            Feb 14 '16 at 4:25






                          • 12




                            $begingroup$
                            @user230452 Unfortunately not! Stems from the fact that $text{arcsinh}{frac{1}{2}}=logvarphi$, and this connection comes by noting that $x^2-x-1=0implies frac{x-frac{1}{x}}{2}=frac{1}{2}$
                            $endgroup$
                            – πr8
                            Feb 14 '16 at 4:28








                          • 3




                            $begingroup$
                            What about $$int_0^{1/2}left(frac{x}{sqrt{x^2+1}}+3right),dx$$
                            $endgroup$
                            – Yves Daoust
                            Feb 14 '16 at 18:02








                          • 48




                            $begingroup$
                            +1 for the understatement, the neat answer and the awesome username. I assume you greet other $pi r8$s by saying "$Ar^k$" for some $kgeq2$.
                            $endgroup$
                            – David Richerby
                            Feb 14 '16 at 18:23






                          • 5




                            $begingroup$
                            @DavidRicherby Indeed - though I'm humbled enough by the reception this first integral seems to have received that I might be well-advised to go by $varphi$r$8$ from here onwards ^^.
                            $endgroup$
                            – πr8
                            Feb 16 '16 at 14:00


















                          • $begingroup$
                            Wow. Did you come up with this by yourself ?
                            $endgroup$
                            – user230452
                            Feb 14 '16 at 4:25






                          • 12




                            $begingroup$
                            @user230452 Unfortunately not! Stems from the fact that $text{arcsinh}{frac{1}{2}}=logvarphi$, and this connection comes by noting that $x^2-x-1=0implies frac{x-frac{1}{x}}{2}=frac{1}{2}$
                            $endgroup$
                            – πr8
                            Feb 14 '16 at 4:28








                          • 3




                            $begingroup$
                            What about $$int_0^{1/2}left(frac{x}{sqrt{x^2+1}}+3right),dx$$
                            $endgroup$
                            – Yves Daoust
                            Feb 14 '16 at 18:02








                          • 48




                            $begingroup$
                            +1 for the understatement, the neat answer and the awesome username. I assume you greet other $pi r8$s by saying "$Ar^k$" for some $kgeq2$.
                            $endgroup$
                            – David Richerby
                            Feb 14 '16 at 18:23






                          • 5




                            $begingroup$
                            @DavidRicherby Indeed - though I'm humbled enough by the reception this first integral seems to have received that I might be well-advised to go by $varphi$r$8$ from here onwards ^^.
                            $endgroup$
                            – πr8
                            Feb 16 '16 at 14:00
















                          $begingroup$
                          Wow. Did you come up with this by yourself ?
                          $endgroup$
                          – user230452
                          Feb 14 '16 at 4:25




                          $begingroup$
                          Wow. Did you come up with this by yourself ?
                          $endgroup$
                          – user230452
                          Feb 14 '16 at 4:25




                          12




                          12




                          $begingroup$
                          @user230452 Unfortunately not! Stems from the fact that $text{arcsinh}{frac{1}{2}}=logvarphi$, and this connection comes by noting that $x^2-x-1=0implies frac{x-frac{1}{x}}{2}=frac{1}{2}$
                          $endgroup$
                          – πr8
                          Feb 14 '16 at 4:28






                          $begingroup$
                          @user230452 Unfortunately not! Stems from the fact that $text{arcsinh}{frac{1}{2}}=logvarphi$, and this connection comes by noting that $x^2-x-1=0implies frac{x-frac{1}{x}}{2}=frac{1}{2}$
                          $endgroup$
                          – πr8
                          Feb 14 '16 at 4:28






                          3




                          3




                          $begingroup$
                          What about $$int_0^{1/2}left(frac{x}{sqrt{x^2+1}}+3right),dx$$
                          $endgroup$
                          – Yves Daoust
                          Feb 14 '16 at 18:02






                          $begingroup$
                          What about $$int_0^{1/2}left(frac{x}{sqrt{x^2+1}}+3right),dx$$
                          $endgroup$
                          – Yves Daoust
                          Feb 14 '16 at 18:02






                          48




                          48




                          $begingroup$
                          +1 for the understatement, the neat answer and the awesome username. I assume you greet other $pi r8$s by saying "$Ar^k$" for some $kgeq2$.
                          $endgroup$
                          – David Richerby
                          Feb 14 '16 at 18:23




                          $begingroup$
                          +1 for the understatement, the neat answer and the awesome username. I assume you greet other $pi r8$s by saying "$Ar^k$" for some $kgeq2$.
                          $endgroup$
                          – David Richerby
                          Feb 14 '16 at 18:23




                          5




                          5




                          $begingroup$
                          @DavidRicherby Indeed - though I'm humbled enough by the reception this first integral seems to have received that I might be well-advised to go by $varphi$r$8$ from here onwards ^^.
                          $endgroup$
                          – πr8
                          Feb 16 '16 at 14:00




                          $begingroup$
                          @DavidRicherby Indeed - though I'm humbled enough by the reception this first integral seems to have received that I might be well-advised to go by $varphi$r$8$ from here onwards ^^.
                          $endgroup$
                          – πr8
                          Feb 16 '16 at 14:00











                          77












                          $begingroup$

                          In this answer, it is shown that
                          $$
                          int_0^inftyfrac{sqrt{x}}{x^2+2x+5}mathrm{d}x=fracpi{2sqrtphi}
                          $$






                          share|cite|improve this answer











                          $endgroup$









                          • 4




                            $begingroup$
                            Awesome!! A strict link between $pi$ and $phi$, I love those things. Thank you!
                            $endgroup$
                            – Von Neumann
                            Feb 14 '16 at 14:35










                          • $begingroup$
                            Brilliant!! Absolutely amazing
                            $endgroup$
                            – Albas
                            Feb 14 '16 at 15:04










                          • $begingroup$
                            wow! this is incredible
                            $endgroup$
                            – Andres Mejia
                            Feb 14 '16 at 16:45






                          • 2




                            $begingroup$
                            So we know $pi=2eint_0^{infty}{cos(x)over x^2+1}text{d}x$ and $e=sum_{k=0}^{infty}{1over k!}$ from the OP, then this answer says $int_0^infty{sqrt{x}over x^2+2x+5}text{d}x={piover 2sqrt{Phi}}$. My immediate thought was to combine the above to get $Phi=left({sum_{k=0}^{infty}{1over k!}int_0^{infty}{cos(x)over x^2+1}text{d}x over int_0^infty{sqrt{x}over x^2+2x+5}text{d}x}right)^2$, which might be considered "interesting".
                            $endgroup$
                            – MichaelS
                            Feb 14 '16 at 22:36










                          • $begingroup$
                            So very nice ! Somehow you perhaps can rope in $e$ too.
                            $endgroup$
                            – Narasimham
                            Feb 15 '16 at 15:18
















                          77












                          $begingroup$

                          In this answer, it is shown that
                          $$
                          int_0^inftyfrac{sqrt{x}}{x^2+2x+5}mathrm{d}x=fracpi{2sqrtphi}
                          $$






                          share|cite|improve this answer











                          $endgroup$









                          • 4




                            $begingroup$
                            Awesome!! A strict link between $pi$ and $phi$, I love those things. Thank you!
                            $endgroup$
                            – Von Neumann
                            Feb 14 '16 at 14:35










                          • $begingroup$
                            Brilliant!! Absolutely amazing
                            $endgroup$
                            – Albas
                            Feb 14 '16 at 15:04










                          • $begingroup$
                            wow! this is incredible
                            $endgroup$
                            – Andres Mejia
                            Feb 14 '16 at 16:45






                          • 2




                            $begingroup$
                            So we know $pi=2eint_0^{infty}{cos(x)over x^2+1}text{d}x$ and $e=sum_{k=0}^{infty}{1over k!}$ from the OP, then this answer says $int_0^infty{sqrt{x}over x^2+2x+5}text{d}x={piover 2sqrt{Phi}}$. My immediate thought was to combine the above to get $Phi=left({sum_{k=0}^{infty}{1over k!}int_0^{infty}{cos(x)over x^2+1}text{d}x over int_0^infty{sqrt{x}over x^2+2x+5}text{d}x}right)^2$, which might be considered "interesting".
                            $endgroup$
                            – MichaelS
                            Feb 14 '16 at 22:36










                          • $begingroup$
                            So very nice ! Somehow you perhaps can rope in $e$ too.
                            $endgroup$
                            – Narasimham
                            Feb 15 '16 at 15:18














                          77












                          77








                          77





                          $begingroup$

                          In this answer, it is shown that
                          $$
                          int_0^inftyfrac{sqrt{x}}{x^2+2x+5}mathrm{d}x=fracpi{2sqrtphi}
                          $$






                          share|cite|improve this answer











                          $endgroup$



                          In this answer, it is shown that
                          $$
                          int_0^inftyfrac{sqrt{x}}{x^2+2x+5}mathrm{d}x=fracpi{2sqrtphi}
                          $$







                          share|cite|improve this answer














                          share|cite|improve this answer



                          share|cite|improve this answer








                          edited Apr 13 '17 at 12:21









                          Community

                          1




                          1










                          answered Feb 14 '16 at 14:33









                          robjohnrobjohn

                          266k27306630




                          266k27306630








                          • 4




                            $begingroup$
                            Awesome!! A strict link between $pi$ and $phi$, I love those things. Thank you!
                            $endgroup$
                            – Von Neumann
                            Feb 14 '16 at 14:35










                          • $begingroup$
                            Brilliant!! Absolutely amazing
                            $endgroup$
                            – Albas
                            Feb 14 '16 at 15:04










                          • $begingroup$
                            wow! this is incredible
                            $endgroup$
                            – Andres Mejia
                            Feb 14 '16 at 16:45






                          • 2




                            $begingroup$
                            So we know $pi=2eint_0^{infty}{cos(x)over x^2+1}text{d}x$ and $e=sum_{k=0}^{infty}{1over k!}$ from the OP, then this answer says $int_0^infty{sqrt{x}over x^2+2x+5}text{d}x={piover 2sqrt{Phi}}$. My immediate thought was to combine the above to get $Phi=left({sum_{k=0}^{infty}{1over k!}int_0^{infty}{cos(x)over x^2+1}text{d}x over int_0^infty{sqrt{x}over x^2+2x+5}text{d}x}right)^2$, which might be considered "interesting".
                            $endgroup$
                            – MichaelS
                            Feb 14 '16 at 22:36










                          • $begingroup$
                            So very nice ! Somehow you perhaps can rope in $e$ too.
                            $endgroup$
                            – Narasimham
                            Feb 15 '16 at 15:18














                          • 4




                            $begingroup$
                            Awesome!! A strict link between $pi$ and $phi$, I love those things. Thank you!
                            $endgroup$
                            – Von Neumann
                            Feb 14 '16 at 14:35










                          • $begingroup$
                            Brilliant!! Absolutely amazing
                            $endgroup$
                            – Albas
                            Feb 14 '16 at 15:04










                          • $begingroup$
                            wow! this is incredible
                            $endgroup$
                            – Andres Mejia
                            Feb 14 '16 at 16:45






                          • 2




                            $begingroup$
                            So we know $pi=2eint_0^{infty}{cos(x)over x^2+1}text{d}x$ and $e=sum_{k=0}^{infty}{1over k!}$ from the OP, then this answer says $int_0^infty{sqrt{x}over x^2+2x+5}text{d}x={piover 2sqrt{Phi}}$. My immediate thought was to combine the above to get $Phi=left({sum_{k=0}^{infty}{1over k!}int_0^{infty}{cos(x)over x^2+1}text{d}x over int_0^infty{sqrt{x}over x^2+2x+5}text{d}x}right)^2$, which might be considered "interesting".
                            $endgroup$
                            – MichaelS
                            Feb 14 '16 at 22:36










                          • $begingroup$
                            So very nice ! Somehow you perhaps can rope in $e$ too.
                            $endgroup$
                            – Narasimham
                            Feb 15 '16 at 15:18








                          4




                          4




                          $begingroup$
                          Awesome!! A strict link between $pi$ and $phi$, I love those things. Thank you!
                          $endgroup$
                          – Von Neumann
                          Feb 14 '16 at 14:35




                          $begingroup$
                          Awesome!! A strict link between $pi$ and $phi$, I love those things. Thank you!
                          $endgroup$
                          – Von Neumann
                          Feb 14 '16 at 14:35












                          $begingroup$
                          Brilliant!! Absolutely amazing
                          $endgroup$
                          – Albas
                          Feb 14 '16 at 15:04




                          $begingroup$
                          Brilliant!! Absolutely amazing
                          $endgroup$
                          – Albas
                          Feb 14 '16 at 15:04












                          $begingroup$
                          wow! this is incredible
                          $endgroup$
                          – Andres Mejia
                          Feb 14 '16 at 16:45




                          $begingroup$
                          wow! this is incredible
                          $endgroup$
                          – Andres Mejia
                          Feb 14 '16 at 16:45




                          2




                          2




                          $begingroup$
                          So we know $pi=2eint_0^{infty}{cos(x)over x^2+1}text{d}x$ and $e=sum_{k=0}^{infty}{1over k!}$ from the OP, then this answer says $int_0^infty{sqrt{x}over x^2+2x+5}text{d}x={piover 2sqrt{Phi}}$. My immediate thought was to combine the above to get $Phi=left({sum_{k=0}^{infty}{1over k!}int_0^{infty}{cos(x)over x^2+1}text{d}x over int_0^infty{sqrt{x}over x^2+2x+5}text{d}x}right)^2$, which might be considered "interesting".
                          $endgroup$
                          – MichaelS
                          Feb 14 '16 at 22:36




                          $begingroup$
                          So we know $pi=2eint_0^{infty}{cos(x)over x^2+1}text{d}x$ and $e=sum_{k=0}^{infty}{1over k!}$ from the OP, then this answer says $int_0^infty{sqrt{x}over x^2+2x+5}text{d}x={piover 2sqrt{Phi}}$. My immediate thought was to combine the above to get $Phi=left({sum_{k=0}^{infty}{1over k!}int_0^{infty}{cos(x)over x^2+1}text{d}x over int_0^infty{sqrt{x}over x^2+2x+5}text{d}x}right)^2$, which might be considered "interesting".
                          $endgroup$
                          – MichaelS
                          Feb 14 '16 at 22:36












                          $begingroup$
                          So very nice ! Somehow you perhaps can rope in $e$ too.
                          $endgroup$
                          – Narasimham
                          Feb 15 '16 at 15:18




                          $begingroup$
                          So very nice ! Somehow you perhaps can rope in $e$ too.
                          $endgroup$
                          – Narasimham
                          Feb 15 '16 at 15:18











                          48












                          $begingroup$

                          [EDIT:20190112 Another connection between $pi$ and $phi$ at the bottom] An identity derived from the Rogers-Ramanujan continued fraction ($R(q)$, not defined here) exhibits a $phi$ factor:



                          $$ frac{1}{(sqrt{phisqrt{5}})e^{2pi/5}} = 1+frac{e^{-2pi}}{1+frac{e^{-4pi}}{1+frac{e^{-6pi}}{1+frac{e^{-8pi}}{1+frac{e^{-10pi}}{1+frac{e^{-12pi}}{cdots}}}}}}$$



                          artist view of the identity



                          and one can then obtain a formula like:
                          $$ ln left( sqrt{4phi+3}-phi^2right) = -frac{1}{5}int_{e^{-2pi}}^1 frac{(1-t)^5(1-t^2)^5(1-t^3)^5 dots}{(1-t^5)(1-t^{10})(1-t^{15}) dots}frac{dt}{t}$$
                          which beautifully links integrals, $e$, $phi$ and $pi$. It is described for instance in Golden Ratio and a Ramanujan-Type Integral.



                          Not very practical though to obtain $phi$ rational approximations.



                          [EDIT] In M. D. Hirschhorn, A connection between $pi$ and $phi$, Fibonacci Quarterly, 2015, another asymptotic relation is:



                          $$ frac{1}{pi}=lim_{nto infty} 2n {5}^{1/4}sum_{k=0}^{n}binom{n}{k}^2binom{n+k}{k}/phi^{5n+5/2}$$






                          share|cite|improve this answer











                          $endgroup$









                          • 15




                            $begingroup$
                            The genius of Ramanujan will always remain a mystery.. what a genius.
                            $endgroup$
                            – Von Neumann
                            Feb 15 '16 at 14:33






                          • 8




                            $begingroup$
                            And I believe it is a good thing that this remains a mystery.
                            $endgroup$
                            – Laurent Duval
                            Feb 15 '16 at 14:58






                          • 1




                            $begingroup$
                            mind... blown...
                            $endgroup$
                            – MichaelChirico
                            Feb 18 '16 at 5:03










                          • $begingroup$
                            @FourierTransform right!
                            $endgroup$
                            – Fawad
                            Oct 8 '16 at 9:14










                          • $begingroup$
                            The link seems dead, here is an archived version: pdf, html.
                            $endgroup$
                            – Vladimir Reshetnikov
                            Jan 31 '18 at 3:08
















                          48












                          $begingroup$

                          [EDIT:20190112 Another connection between $pi$ and $phi$ at the bottom] An identity derived from the Rogers-Ramanujan continued fraction ($R(q)$, not defined here) exhibits a $phi$ factor:



                          $$ frac{1}{(sqrt{phisqrt{5}})e^{2pi/5}} = 1+frac{e^{-2pi}}{1+frac{e^{-4pi}}{1+frac{e^{-6pi}}{1+frac{e^{-8pi}}{1+frac{e^{-10pi}}{1+frac{e^{-12pi}}{cdots}}}}}}$$



                          artist view of the identity



                          and one can then obtain a formula like:
                          $$ ln left( sqrt{4phi+3}-phi^2right) = -frac{1}{5}int_{e^{-2pi}}^1 frac{(1-t)^5(1-t^2)^5(1-t^3)^5 dots}{(1-t^5)(1-t^{10})(1-t^{15}) dots}frac{dt}{t}$$
                          which beautifully links integrals, $e$, $phi$ and $pi$. It is described for instance in Golden Ratio and a Ramanujan-Type Integral.



                          Not very practical though to obtain $phi$ rational approximations.



                          [EDIT] In M. D. Hirschhorn, A connection between $pi$ and $phi$, Fibonacci Quarterly, 2015, another asymptotic relation is:



                          $$ frac{1}{pi}=lim_{nto infty} 2n {5}^{1/4}sum_{k=0}^{n}binom{n}{k}^2binom{n+k}{k}/phi^{5n+5/2}$$






                          share|cite|improve this answer











                          $endgroup$









                          • 15




                            $begingroup$
                            The genius of Ramanujan will always remain a mystery.. what a genius.
                            $endgroup$
                            – Von Neumann
                            Feb 15 '16 at 14:33






                          • 8




                            $begingroup$
                            And I believe it is a good thing that this remains a mystery.
                            $endgroup$
                            – Laurent Duval
                            Feb 15 '16 at 14:58






                          • 1




                            $begingroup$
                            mind... blown...
                            $endgroup$
                            – MichaelChirico
                            Feb 18 '16 at 5:03










                          • $begingroup$
                            @FourierTransform right!
                            $endgroup$
                            – Fawad
                            Oct 8 '16 at 9:14










                          • $begingroup$
                            The link seems dead, here is an archived version: pdf, html.
                            $endgroup$
                            – Vladimir Reshetnikov
                            Jan 31 '18 at 3:08














                          48












                          48








                          48





                          $begingroup$

                          [EDIT:20190112 Another connection between $pi$ and $phi$ at the bottom] An identity derived from the Rogers-Ramanujan continued fraction ($R(q)$, not defined here) exhibits a $phi$ factor:



                          $$ frac{1}{(sqrt{phisqrt{5}})e^{2pi/5}} = 1+frac{e^{-2pi}}{1+frac{e^{-4pi}}{1+frac{e^{-6pi}}{1+frac{e^{-8pi}}{1+frac{e^{-10pi}}{1+frac{e^{-12pi}}{cdots}}}}}}$$



                          artist view of the identity



                          and one can then obtain a formula like:
                          $$ ln left( sqrt{4phi+3}-phi^2right) = -frac{1}{5}int_{e^{-2pi}}^1 frac{(1-t)^5(1-t^2)^5(1-t^3)^5 dots}{(1-t^5)(1-t^{10})(1-t^{15}) dots}frac{dt}{t}$$
                          which beautifully links integrals, $e$, $phi$ and $pi$. It is described for instance in Golden Ratio and a Ramanujan-Type Integral.



                          Not very practical though to obtain $phi$ rational approximations.



                          [EDIT] In M. D. Hirschhorn, A connection between $pi$ and $phi$, Fibonacci Quarterly, 2015, another asymptotic relation is:



                          $$ frac{1}{pi}=lim_{nto infty} 2n {5}^{1/4}sum_{k=0}^{n}binom{n}{k}^2binom{n+k}{k}/phi^{5n+5/2}$$






                          share|cite|improve this answer











                          $endgroup$



                          [EDIT:20190112 Another connection between $pi$ and $phi$ at the bottom] An identity derived from the Rogers-Ramanujan continued fraction ($R(q)$, not defined here) exhibits a $phi$ factor:



                          $$ frac{1}{(sqrt{phisqrt{5}})e^{2pi/5}} = 1+frac{e^{-2pi}}{1+frac{e^{-4pi}}{1+frac{e^{-6pi}}{1+frac{e^{-8pi}}{1+frac{e^{-10pi}}{1+frac{e^{-12pi}}{cdots}}}}}}$$



                          artist view of the identity



                          and one can then obtain a formula like:
                          $$ ln left( sqrt{4phi+3}-phi^2right) = -frac{1}{5}int_{e^{-2pi}}^1 frac{(1-t)^5(1-t^2)^5(1-t^3)^5 dots}{(1-t^5)(1-t^{10})(1-t^{15}) dots}frac{dt}{t}$$
                          which beautifully links integrals, $e$, $phi$ and $pi$. It is described for instance in Golden Ratio and a Ramanujan-Type Integral.



                          Not very practical though to obtain $phi$ rational approximations.



                          [EDIT] In M. D. Hirschhorn, A connection between $pi$ and $phi$, Fibonacci Quarterly, 2015, another asymptotic relation is:



                          $$ frac{1}{pi}=lim_{nto infty} 2n {5}^{1/4}sum_{k=0}^{n}binom{n}{k}^2binom{n+k}{k}/phi^{5n+5/2}$$







                          share|cite|improve this answer














                          share|cite|improve this answer



                          share|cite|improve this answer








                          edited Jan 12 at 16:42

























                          answered Feb 14 '16 at 8:22









                          Laurent DuvalLaurent Duval

                          5,34311240




                          5,34311240








                          • 15




                            $begingroup$
                            The genius of Ramanujan will always remain a mystery.. what a genius.
                            $endgroup$
                            – Von Neumann
                            Feb 15 '16 at 14:33






                          • 8




                            $begingroup$
                            And I believe it is a good thing that this remains a mystery.
                            $endgroup$
                            – Laurent Duval
                            Feb 15 '16 at 14:58






                          • 1




                            $begingroup$
                            mind... blown...
                            $endgroup$
                            – MichaelChirico
                            Feb 18 '16 at 5:03










                          • $begingroup$
                            @FourierTransform right!
                            $endgroup$
                            – Fawad
                            Oct 8 '16 at 9:14










                          • $begingroup$
                            The link seems dead, here is an archived version: pdf, html.
                            $endgroup$
                            – Vladimir Reshetnikov
                            Jan 31 '18 at 3:08














                          • 15




                            $begingroup$
                            The genius of Ramanujan will always remain a mystery.. what a genius.
                            $endgroup$
                            – Von Neumann
                            Feb 15 '16 at 14:33






                          • 8




                            $begingroup$
                            And I believe it is a good thing that this remains a mystery.
                            $endgroup$
                            – Laurent Duval
                            Feb 15 '16 at 14:58






                          • 1




                            $begingroup$
                            mind... blown...
                            $endgroup$
                            – MichaelChirico
                            Feb 18 '16 at 5:03










                          • $begingroup$
                            @FourierTransform right!
                            $endgroup$
                            – Fawad
                            Oct 8 '16 at 9:14










                          • $begingroup$
                            The link seems dead, here is an archived version: pdf, html.
                            $endgroup$
                            – Vladimir Reshetnikov
                            Jan 31 '18 at 3:08








                          15




                          15




                          $begingroup$
                          The genius of Ramanujan will always remain a mystery.. what a genius.
                          $endgroup$
                          – Von Neumann
                          Feb 15 '16 at 14:33




                          $begingroup$
                          The genius of Ramanujan will always remain a mystery.. what a genius.
                          $endgroup$
                          – Von Neumann
                          Feb 15 '16 at 14:33




                          8




                          8




                          $begingroup$
                          And I believe it is a good thing that this remains a mystery.
                          $endgroup$
                          – Laurent Duval
                          Feb 15 '16 at 14:58




                          $begingroup$
                          And I believe it is a good thing that this remains a mystery.
                          $endgroup$
                          – Laurent Duval
                          Feb 15 '16 at 14:58




                          1




                          1




                          $begingroup$
                          mind... blown...
                          $endgroup$
                          – MichaelChirico
                          Feb 18 '16 at 5:03




                          $begingroup$
                          mind... blown...
                          $endgroup$
                          – MichaelChirico
                          Feb 18 '16 at 5:03












                          $begingroup$
                          @FourierTransform right!
                          $endgroup$
                          – Fawad
                          Oct 8 '16 at 9:14




                          $begingroup$
                          @FourierTransform right!
                          $endgroup$
                          – Fawad
                          Oct 8 '16 at 9:14












                          $begingroup$
                          The link seems dead, here is an archived version: pdf, html.
                          $endgroup$
                          – Vladimir Reshetnikov
                          Jan 31 '18 at 3:08




                          $begingroup$
                          The link seems dead, here is an archived version: pdf, html.
                          $endgroup$
                          – Vladimir Reshetnikov
                          Jan 31 '18 at 3:08











                          33












                          $begingroup$

                          $$int_{-1}^1 dx frac1x sqrt{frac{1+x}{1-x}} log{left (frac{2 x^2+2 x+1}{2 x^2-2 x+1}right )} = 4 pi operatorname{arccot}{sqrt{phi}}$$






                          share|cite|improve this answer











                          $endgroup$













                          • $begingroup$
                            There is a sign error in the log term
                            $endgroup$
                            – Cyclohexanol.
                            Feb 14 '16 at 16:37










                          • $begingroup$
                            @LaplacianFourier: Thanks.
                            $endgroup$
                            – Ron Gordon
                            Feb 14 '16 at 16:37






                          • 4




                            $begingroup$
                            Ah yes, isn't this like the most upvoted post on this site? Always fun reading even though I don't know enough math do do it..
                            $endgroup$
                            – Faraz Masroor
                            Feb 14 '16 at 21:48










                          • $begingroup$
                            @FarazMasroor: Actually, I think the 7th or 8th-most upvoted post. But thanks - if you want to learn feel free to ask questions!
                            $endgroup$
                            – Ron Gordon
                            Feb 14 '16 at 21:51










                          • $begingroup$
                            ...might as well include a link: MSE 562964
                            $endgroup$
                            – Benjamin Dickman
                            Feb 18 '16 at 7:27
















                          33












                          $begingroup$

                          $$int_{-1}^1 dx frac1x sqrt{frac{1+x}{1-x}} log{left (frac{2 x^2+2 x+1}{2 x^2-2 x+1}right )} = 4 pi operatorname{arccot}{sqrt{phi}}$$






                          share|cite|improve this answer











                          $endgroup$













                          • $begingroup$
                            There is a sign error in the log term
                            $endgroup$
                            – Cyclohexanol.
                            Feb 14 '16 at 16:37










                          • $begingroup$
                            @LaplacianFourier: Thanks.
                            $endgroup$
                            – Ron Gordon
                            Feb 14 '16 at 16:37






                          • 4




                            $begingroup$
                            Ah yes, isn't this like the most upvoted post on this site? Always fun reading even though I don't know enough math do do it..
                            $endgroup$
                            – Faraz Masroor
                            Feb 14 '16 at 21:48










                          • $begingroup$
                            @FarazMasroor: Actually, I think the 7th or 8th-most upvoted post. But thanks - if you want to learn feel free to ask questions!
                            $endgroup$
                            – Ron Gordon
                            Feb 14 '16 at 21:51










                          • $begingroup$
                            ...might as well include a link: MSE 562964
                            $endgroup$
                            – Benjamin Dickman
                            Feb 18 '16 at 7:27














                          33












                          33








                          33





                          $begingroup$

                          $$int_{-1}^1 dx frac1x sqrt{frac{1+x}{1-x}} log{left (frac{2 x^2+2 x+1}{2 x^2-2 x+1}right )} = 4 pi operatorname{arccot}{sqrt{phi}}$$






                          share|cite|improve this answer











                          $endgroup$



                          $$int_{-1}^1 dx frac1x sqrt{frac{1+x}{1-x}} log{left (frac{2 x^2+2 x+1}{2 x^2-2 x+1}right )} = 4 pi operatorname{arccot}{sqrt{phi}}$$







                          share|cite|improve this answer














                          share|cite|improve this answer



                          share|cite|improve this answer








                          edited Feb 14 '16 at 16:37

























                          answered Feb 14 '16 at 16:18









                          Ron GordonRon Gordon

                          122k14154264




                          122k14154264












                          • $begingroup$
                            There is a sign error in the log term
                            $endgroup$
                            – Cyclohexanol.
                            Feb 14 '16 at 16:37










                          • $begingroup$
                            @LaplacianFourier: Thanks.
                            $endgroup$
                            – Ron Gordon
                            Feb 14 '16 at 16:37






                          • 4




                            $begingroup$
                            Ah yes, isn't this like the most upvoted post on this site? Always fun reading even though I don't know enough math do do it..
                            $endgroup$
                            – Faraz Masroor
                            Feb 14 '16 at 21:48










                          • $begingroup$
                            @FarazMasroor: Actually, I think the 7th or 8th-most upvoted post. But thanks - if you want to learn feel free to ask questions!
                            $endgroup$
                            – Ron Gordon
                            Feb 14 '16 at 21:51










                          • $begingroup$
                            ...might as well include a link: MSE 562964
                            $endgroup$
                            – Benjamin Dickman
                            Feb 18 '16 at 7:27


















                          • $begingroup$
                            There is a sign error in the log term
                            $endgroup$
                            – Cyclohexanol.
                            Feb 14 '16 at 16:37










                          • $begingroup$
                            @LaplacianFourier: Thanks.
                            $endgroup$
                            – Ron Gordon
                            Feb 14 '16 at 16:37






                          • 4




                            $begingroup$
                            Ah yes, isn't this like the most upvoted post on this site? Always fun reading even though I don't know enough math do do it..
                            $endgroup$
                            – Faraz Masroor
                            Feb 14 '16 at 21:48










                          • $begingroup$
                            @FarazMasroor: Actually, I think the 7th or 8th-most upvoted post. But thanks - if you want to learn feel free to ask questions!
                            $endgroup$
                            – Ron Gordon
                            Feb 14 '16 at 21:51










                          • $begingroup$
                            ...might as well include a link: MSE 562964
                            $endgroup$
                            – Benjamin Dickman
                            Feb 18 '16 at 7:27
















                          $begingroup$
                          There is a sign error in the log term
                          $endgroup$
                          – Cyclohexanol.
                          Feb 14 '16 at 16:37




                          $begingroup$
                          There is a sign error in the log term
                          $endgroup$
                          – Cyclohexanol.
                          Feb 14 '16 at 16:37












                          $begingroup$
                          @LaplacianFourier: Thanks.
                          $endgroup$
                          – Ron Gordon
                          Feb 14 '16 at 16:37




                          $begingroup$
                          @LaplacianFourier: Thanks.
                          $endgroup$
                          – Ron Gordon
                          Feb 14 '16 at 16:37




                          4




                          4




                          $begingroup$
                          Ah yes, isn't this like the most upvoted post on this site? Always fun reading even though I don't know enough math do do it..
                          $endgroup$
                          – Faraz Masroor
                          Feb 14 '16 at 21:48




                          $begingroup$
                          Ah yes, isn't this like the most upvoted post on this site? Always fun reading even though I don't know enough math do do it..
                          $endgroup$
                          – Faraz Masroor
                          Feb 14 '16 at 21:48












                          $begingroup$
                          @FarazMasroor: Actually, I think the 7th or 8th-most upvoted post. But thanks - if you want to learn feel free to ask questions!
                          $endgroup$
                          – Ron Gordon
                          Feb 14 '16 at 21:51




                          $begingroup$
                          @FarazMasroor: Actually, I think the 7th or 8th-most upvoted post. But thanks - if you want to learn feel free to ask questions!
                          $endgroup$
                          – Ron Gordon
                          Feb 14 '16 at 21:51












                          $begingroup$
                          ...might as well include a link: MSE 562964
                          $endgroup$
                          – Benjamin Dickman
                          Feb 18 '16 at 7:27




                          $begingroup$
                          ...might as well include a link: MSE 562964
                          $endgroup$
                          – Benjamin Dickman
                          Feb 18 '16 at 7:27











                          31












                          $begingroup$

                          Here's a series:



                          $$
                          phi = 1 + sum_{n=2}^infty frac{(-1)^{n}}{F_nF_{n-1}}
                          $$



                          where $F_n$ is the $n$th Fibonacci number.



                          To see this, rewrite the numerator using the identity $(-1)^n=F_{n+1}F_{n-1}-F_n^2$, at which point the summand becomes
                          $$
                          frac{F_{n+1}F_{n-1}-F_n^2}{F_nF_{n-1}}=frac{F_{n+1}}{F_n}-frac{F_n}{F_{n-1}}
                          $$
                          and so the sum telescopes: the partial sum ending at $n$ is equal to
                          $$
                          frac{F_{n+1}}{F_n}-frac{F_2}{F_1}=frac{F_{n+1}}{F_n} - 1
                          $$
                          which gives the original expression for the series via the limit $lim_{n to infty} frac{F_{n+1}}{F_n} = phi$.






                          share|cite|improve this answer









                          $endgroup$













                          • $begingroup$
                            Was this the first definition of golden ratio or did it have a definition before that ?
                            $endgroup$
                            – user230452
                            Feb 14 '16 at 4:27












                          • $begingroup$
                            @user230452 $phi = frac { 1+ sqrt 5}2$
                            $endgroup$
                            – Ant
                            Feb 14 '16 at 9:43










                          • $begingroup$
                            I mean, didn't that number come from the Fibonacci series itself or did it already have a definition and was found again in the Fibonacci series ?
                            $endgroup$
                            – user230452
                            Feb 14 '16 at 10:23






                          • 8




                            $begingroup$
                            @user230452 The golden ratio first arose as a ratio between quantities $a,b$ for which ratio $a:b$ is the same as ratio $a+b:a$. Hence, it came before (or at least independently of) Fibonacci numbers.
                            $endgroup$
                            – Wojowu
                            Feb 14 '16 at 10:32






                          • 3




                            $begingroup$
                            @Wojowu My point was just that the Fibonacci sequence is all about situations where you're dealing with $a$, $b$ and $a+b$, and so is the classical definition of the golden ratio. Whereas, for example, $(1+sqrt{5})/2$ is a completely different way of defining the same number. Anyway, I'm just nit-picking.
                            $endgroup$
                            – David Richerby
                            Feb 14 '16 at 21:26
















                          31












                          $begingroup$

                          Here's a series:



                          $$
                          phi = 1 + sum_{n=2}^infty frac{(-1)^{n}}{F_nF_{n-1}}
                          $$



                          where $F_n$ is the $n$th Fibonacci number.



                          To see this, rewrite the numerator using the identity $(-1)^n=F_{n+1}F_{n-1}-F_n^2$, at which point the summand becomes
                          $$
                          frac{F_{n+1}F_{n-1}-F_n^2}{F_nF_{n-1}}=frac{F_{n+1}}{F_n}-frac{F_n}{F_{n-1}}
                          $$
                          and so the sum telescopes: the partial sum ending at $n$ is equal to
                          $$
                          frac{F_{n+1}}{F_n}-frac{F_2}{F_1}=frac{F_{n+1}}{F_n} - 1
                          $$
                          which gives the original expression for the series via the limit $lim_{n to infty} frac{F_{n+1}}{F_n} = phi$.






                          share|cite|improve this answer









                          $endgroup$













                          • $begingroup$
                            Was this the first definition of golden ratio or did it have a definition before that ?
                            $endgroup$
                            – user230452
                            Feb 14 '16 at 4:27












                          • $begingroup$
                            @user230452 $phi = frac { 1+ sqrt 5}2$
                            $endgroup$
                            – Ant
                            Feb 14 '16 at 9:43










                          • $begingroup$
                            I mean, didn't that number come from the Fibonacci series itself or did it already have a definition and was found again in the Fibonacci series ?
                            $endgroup$
                            – user230452
                            Feb 14 '16 at 10:23






                          • 8




                            $begingroup$
                            @user230452 The golden ratio first arose as a ratio between quantities $a,b$ for which ratio $a:b$ is the same as ratio $a+b:a$. Hence, it came before (or at least independently of) Fibonacci numbers.
                            $endgroup$
                            – Wojowu
                            Feb 14 '16 at 10:32






                          • 3




                            $begingroup$
                            @Wojowu My point was just that the Fibonacci sequence is all about situations where you're dealing with $a$, $b$ and $a+b$, and so is the classical definition of the golden ratio. Whereas, for example, $(1+sqrt{5})/2$ is a completely different way of defining the same number. Anyway, I'm just nit-picking.
                            $endgroup$
                            – David Richerby
                            Feb 14 '16 at 21:26














                          31












                          31








                          31





                          $begingroup$

                          Here's a series:



                          $$
                          phi = 1 + sum_{n=2}^infty frac{(-1)^{n}}{F_nF_{n-1}}
                          $$



                          where $F_n$ is the $n$th Fibonacci number.



                          To see this, rewrite the numerator using the identity $(-1)^n=F_{n+1}F_{n-1}-F_n^2$, at which point the summand becomes
                          $$
                          frac{F_{n+1}F_{n-1}-F_n^2}{F_nF_{n-1}}=frac{F_{n+1}}{F_n}-frac{F_n}{F_{n-1}}
                          $$
                          and so the sum telescopes: the partial sum ending at $n$ is equal to
                          $$
                          frac{F_{n+1}}{F_n}-frac{F_2}{F_1}=frac{F_{n+1}}{F_n} - 1
                          $$
                          which gives the original expression for the series via the limit $lim_{n to infty} frac{F_{n+1}}{F_n} = phi$.






                          share|cite|improve this answer









                          $endgroup$



                          Here's a series:



                          $$
                          phi = 1 + sum_{n=2}^infty frac{(-1)^{n}}{F_nF_{n-1}}
                          $$



                          where $F_n$ is the $n$th Fibonacci number.



                          To see this, rewrite the numerator using the identity $(-1)^n=F_{n+1}F_{n-1}-F_n^2$, at which point the summand becomes
                          $$
                          frac{F_{n+1}F_{n-1}-F_n^2}{F_nF_{n-1}}=frac{F_{n+1}}{F_n}-frac{F_n}{F_{n-1}}
                          $$
                          and so the sum telescopes: the partial sum ending at $n$ is equal to
                          $$
                          frac{F_{n+1}}{F_n}-frac{F_2}{F_1}=frac{F_{n+1}}{F_n} - 1
                          $$
                          which gives the original expression for the series via the limit $lim_{n to infty} frac{F_{n+1}}{F_n} = phi$.







                          share|cite|improve this answer












                          share|cite|improve this answer



                          share|cite|improve this answer










                          answered Feb 14 '16 at 3:29









                          MicahMicah

                          29.9k1364106




                          29.9k1364106












                          • $begingroup$
                            Was this the first definition of golden ratio or did it have a definition before that ?
                            $endgroup$
                            – user230452
                            Feb 14 '16 at 4:27












                          • $begingroup$
                            @user230452 $phi = frac { 1+ sqrt 5}2$
                            $endgroup$
                            – Ant
                            Feb 14 '16 at 9:43










                          • $begingroup$
                            I mean, didn't that number come from the Fibonacci series itself or did it already have a definition and was found again in the Fibonacci series ?
                            $endgroup$
                            – user230452
                            Feb 14 '16 at 10:23






                          • 8




                            $begingroup$
                            @user230452 The golden ratio first arose as a ratio between quantities $a,b$ for which ratio $a:b$ is the same as ratio $a+b:a$. Hence, it came before (or at least independently of) Fibonacci numbers.
                            $endgroup$
                            – Wojowu
                            Feb 14 '16 at 10:32






                          • 3




                            $begingroup$
                            @Wojowu My point was just that the Fibonacci sequence is all about situations where you're dealing with $a$, $b$ and $a+b$, and so is the classical definition of the golden ratio. Whereas, for example, $(1+sqrt{5})/2$ is a completely different way of defining the same number. Anyway, I'm just nit-picking.
                            $endgroup$
                            – David Richerby
                            Feb 14 '16 at 21:26


















                          • $begingroup$
                            Was this the first definition of golden ratio or did it have a definition before that ?
                            $endgroup$
                            – user230452
                            Feb 14 '16 at 4:27












                          • $begingroup$
                            @user230452 $phi = frac { 1+ sqrt 5}2$
                            $endgroup$
                            – Ant
                            Feb 14 '16 at 9:43










                          • $begingroup$
                            I mean, didn't that number come from the Fibonacci series itself or did it already have a definition and was found again in the Fibonacci series ?
                            $endgroup$
                            – user230452
                            Feb 14 '16 at 10:23






                          • 8




                            $begingroup$
                            @user230452 The golden ratio first arose as a ratio between quantities $a,b$ for which ratio $a:b$ is the same as ratio $a+b:a$. Hence, it came before (or at least independently of) Fibonacci numbers.
                            $endgroup$
                            – Wojowu
                            Feb 14 '16 at 10:32






                          • 3




                            $begingroup$
                            @Wojowu My point was just that the Fibonacci sequence is all about situations where you're dealing with $a$, $b$ and $a+b$, and so is the classical definition of the golden ratio. Whereas, for example, $(1+sqrt{5})/2$ is a completely different way of defining the same number. Anyway, I'm just nit-picking.
                            $endgroup$
                            – David Richerby
                            Feb 14 '16 at 21:26
















                          $begingroup$
                          Was this the first definition of golden ratio or did it have a definition before that ?
                          $endgroup$
                          – user230452
                          Feb 14 '16 at 4:27






                          $begingroup$
                          Was this the first definition of golden ratio or did it have a definition before that ?
                          $endgroup$
                          – user230452
                          Feb 14 '16 at 4:27














                          $begingroup$
                          @user230452 $phi = frac { 1+ sqrt 5}2$
                          $endgroup$
                          – Ant
                          Feb 14 '16 at 9:43




                          $begingroup$
                          @user230452 $phi = frac { 1+ sqrt 5}2$
                          $endgroup$
                          – Ant
                          Feb 14 '16 at 9:43












                          $begingroup$
                          I mean, didn't that number come from the Fibonacci series itself or did it already have a definition and was found again in the Fibonacci series ?
                          $endgroup$
                          – user230452
                          Feb 14 '16 at 10:23




                          $begingroup$
                          I mean, didn't that number come from the Fibonacci series itself or did it already have a definition and was found again in the Fibonacci series ?
                          $endgroup$
                          – user230452
                          Feb 14 '16 at 10:23




                          8




                          8




                          $begingroup$
                          @user230452 The golden ratio first arose as a ratio between quantities $a,b$ for which ratio $a:b$ is the same as ratio $a+b:a$. Hence, it came before (or at least independently of) Fibonacci numbers.
                          $endgroup$
                          – Wojowu
                          Feb 14 '16 at 10:32




                          $begingroup$
                          @user230452 The golden ratio first arose as a ratio between quantities $a,b$ for which ratio $a:b$ is the same as ratio $a+b:a$. Hence, it came before (or at least independently of) Fibonacci numbers.
                          $endgroup$
                          – Wojowu
                          Feb 14 '16 at 10:32




                          3




                          3




                          $begingroup$
                          @Wojowu My point was just that the Fibonacci sequence is all about situations where you're dealing with $a$, $b$ and $a+b$, and so is the classical definition of the golden ratio. Whereas, for example, $(1+sqrt{5})/2$ is a completely different way of defining the same number. Anyway, I'm just nit-picking.
                          $endgroup$
                          – David Richerby
                          Feb 14 '16 at 21:26




                          $begingroup$
                          @Wojowu My point was just that the Fibonacci sequence is all about situations where you're dealing with $a$, $b$ and $a+b$, and so is the classical definition of the golden ratio. Whereas, for example, $(1+sqrt{5})/2$ is a completely different way of defining the same number. Anyway, I'm just nit-picking.
                          $endgroup$
                          – David Richerby
                          Feb 14 '16 at 21:26











                          30












                          $begingroup$

                          Based on the fact that $varphi = frac{1+sqrt{5}}{2}$:




                          $$varphi = int_4^5 frac32+frac1{4sqrt{x}} mathrm{d}x$$




                          Based on the fact that $varphi = 2cos(frac{pi}{5})$:




                          $$varphi = int_{tfrac{pi}{5}}^{tfrac{pi}{2}} 2sin(x) mathrm{d}x$$







                          share|cite|improve this answer









                          $endgroup$













                          • $begingroup$
                            I wanted to do that at first, but thought it wasn't 'interesting' by OP's standards
                            $endgroup$
                            – Yuriy S
                            Feb 14 '16 at 12:31






                          • 3




                            $begingroup$
                            @YuriyS I just took 'not interesting' as 'directly containing $varphi$, or a trivial variation on it'
                            $endgroup$
                            – wythagoras
                            Feb 14 '16 at 12:35










                          • $begingroup$
                            Awesome, the second one is great!!
                            $endgroup$
                            – Von Neumann
                            Feb 15 '16 at 14:34
















                          30












                          $begingroup$

                          Based on the fact that $varphi = frac{1+sqrt{5}}{2}$:




                          $$varphi = int_4^5 frac32+frac1{4sqrt{x}} mathrm{d}x$$




                          Based on the fact that $varphi = 2cos(frac{pi}{5})$:




                          $$varphi = int_{tfrac{pi}{5}}^{tfrac{pi}{2}} 2sin(x) mathrm{d}x$$







                          share|cite|improve this answer









                          $endgroup$













                          • $begingroup$
                            I wanted to do that at first, but thought it wasn't 'interesting' by OP's standards
                            $endgroup$
                            – Yuriy S
                            Feb 14 '16 at 12:31






                          • 3




                            $begingroup$
                            @YuriyS I just took 'not interesting' as 'directly containing $varphi$, or a trivial variation on it'
                            $endgroup$
                            – wythagoras
                            Feb 14 '16 at 12:35










                          • $begingroup$
                            Awesome, the second one is great!!
                            $endgroup$
                            – Von Neumann
                            Feb 15 '16 at 14:34














                          30












                          30








                          30





                          $begingroup$

                          Based on the fact that $varphi = frac{1+sqrt{5}}{2}$:




                          $$varphi = int_4^5 frac32+frac1{4sqrt{x}} mathrm{d}x$$




                          Based on the fact that $varphi = 2cos(frac{pi}{5})$:




                          $$varphi = int_{tfrac{pi}{5}}^{tfrac{pi}{2}} 2sin(x) mathrm{d}x$$







                          share|cite|improve this answer









                          $endgroup$



                          Based on the fact that $varphi = frac{1+sqrt{5}}{2}$:




                          $$varphi = int_4^5 frac32+frac1{4sqrt{x}} mathrm{d}x$$




                          Based on the fact that $varphi = 2cos(frac{pi}{5})$:




                          $$varphi = int_{tfrac{pi}{5}}^{tfrac{pi}{2}} 2sin(x) mathrm{d}x$$








                          share|cite|improve this answer












                          share|cite|improve this answer



                          share|cite|improve this answer










                          answered Feb 14 '16 at 10:22









                          wythagoraswythagoras

                          21.6k444104




                          21.6k444104












                          • $begingroup$
                            I wanted to do that at first, but thought it wasn't 'interesting' by OP's standards
                            $endgroup$
                            – Yuriy S
                            Feb 14 '16 at 12:31






                          • 3




                            $begingroup$
                            @YuriyS I just took 'not interesting' as 'directly containing $varphi$, or a trivial variation on it'
                            $endgroup$
                            – wythagoras
                            Feb 14 '16 at 12:35










                          • $begingroup$
                            Awesome, the second one is great!!
                            $endgroup$
                            – Von Neumann
                            Feb 15 '16 at 14:34


















                          • $begingroup$
                            I wanted to do that at first, but thought it wasn't 'interesting' by OP's standards
                            $endgroup$
                            – Yuriy S
                            Feb 14 '16 at 12:31






                          • 3




                            $begingroup$
                            @YuriyS I just took 'not interesting' as 'directly containing $varphi$, or a trivial variation on it'
                            $endgroup$
                            – wythagoras
                            Feb 14 '16 at 12:35










                          • $begingroup$
                            Awesome, the second one is great!!
                            $endgroup$
                            – Von Neumann
                            Feb 15 '16 at 14:34
















                          $begingroup$
                          I wanted to do that at first, but thought it wasn't 'interesting' by OP's standards
                          $endgroup$
                          – Yuriy S
                          Feb 14 '16 at 12:31




                          $begingroup$
                          I wanted to do that at first, but thought it wasn't 'interesting' by OP's standards
                          $endgroup$
                          – Yuriy S
                          Feb 14 '16 at 12:31




                          3




                          3




                          $begingroup$
                          @YuriyS I just took 'not interesting' as 'directly containing $varphi$, or a trivial variation on it'
                          $endgroup$
                          – wythagoras
                          Feb 14 '16 at 12:35




                          $begingroup$
                          @YuriyS I just took 'not interesting' as 'directly containing $varphi$, or a trivial variation on it'
                          $endgroup$
                          – wythagoras
                          Feb 14 '16 at 12:35












                          $begingroup$
                          Awesome, the second one is great!!
                          $endgroup$
                          – Von Neumann
                          Feb 15 '16 at 14:34




                          $begingroup$
                          Awesome, the second one is great!!
                          $endgroup$
                          – Von Neumann
                          Feb 15 '16 at 14:34











                          26












                          $begingroup$

                          $$int_0^{infty} frac{x^2}{1+x^{10}} , mathrm{d}x = frac{pi}{5 phi}.$$






                          share|cite|improve this answer









                          $endgroup$













                          • $begingroup$
                            Great! Another integral that relates two constants! Thank you!
                            $endgroup$
                            – Von Neumann
                            Feb 14 '16 at 19:37










                          • $begingroup$
                            @KimPeek, there is an infinite number of integrals of this kind
                            $endgroup$
                            – Yuriy S
                            Feb 14 '16 at 19:42






                          • 4




                            $begingroup$
                            @YuriyS The more I see, the happier I am :D
                            $endgroup$
                            – Von Neumann
                            Feb 14 '16 at 20:06






                          • 5




                            $begingroup$
                            Although it adds nothing, I think having $5x^2$ on the left instead of $5$ at the denominator on the right looks even prettier (if possible)
                            $endgroup$
                            – Laurent Duval
                            Feb 15 '16 at 7:14
















                          26












                          $begingroup$

                          $$int_0^{infty} frac{x^2}{1+x^{10}} , mathrm{d}x = frac{pi}{5 phi}.$$






                          share|cite|improve this answer









                          $endgroup$













                          • $begingroup$
                            Great! Another integral that relates two constants! Thank you!
                            $endgroup$
                            – Von Neumann
                            Feb 14 '16 at 19:37










                          • $begingroup$
                            @KimPeek, there is an infinite number of integrals of this kind
                            $endgroup$
                            – Yuriy S
                            Feb 14 '16 at 19:42






                          • 4




                            $begingroup$
                            @YuriyS The more I see, the happier I am :D
                            $endgroup$
                            – Von Neumann
                            Feb 14 '16 at 20:06






                          • 5




                            $begingroup$
                            Although it adds nothing, I think having $5x^2$ on the left instead of $5$ at the denominator on the right looks even prettier (if possible)
                            $endgroup$
                            – Laurent Duval
                            Feb 15 '16 at 7:14














                          26












                          26








                          26





                          $begingroup$

                          $$int_0^{infty} frac{x^2}{1+x^{10}} , mathrm{d}x = frac{pi}{5 phi}.$$






                          share|cite|improve this answer









                          $endgroup$



                          $$int_0^{infty} frac{x^2}{1+x^{10}} , mathrm{d}x = frac{pi}{5 phi}.$$







                          share|cite|improve this answer












                          share|cite|improve this answer



                          share|cite|improve this answer










                          answered Feb 14 '16 at 19:27









                          user314474user314474

                          26122




                          26122












                          • $begingroup$
                            Great! Another integral that relates two constants! Thank you!
                            $endgroup$
                            – Von Neumann
                            Feb 14 '16 at 19:37










                          • $begingroup$
                            @KimPeek, there is an infinite number of integrals of this kind
                            $endgroup$
                            – Yuriy S
                            Feb 14 '16 at 19:42






                          • 4




                            $begingroup$
                            @YuriyS The more I see, the happier I am :D
                            $endgroup$
                            – Von Neumann
                            Feb 14 '16 at 20:06






                          • 5




                            $begingroup$
                            Although it adds nothing, I think having $5x^2$ on the left instead of $5$ at the denominator on the right looks even prettier (if possible)
                            $endgroup$
                            – Laurent Duval
                            Feb 15 '16 at 7:14


















                          • $begingroup$
                            Great! Another integral that relates two constants! Thank you!
                            $endgroup$
                            – Von Neumann
                            Feb 14 '16 at 19:37










                          • $begingroup$
                            @KimPeek, there is an infinite number of integrals of this kind
                            $endgroup$
                            – Yuriy S
                            Feb 14 '16 at 19:42






                          • 4




                            $begingroup$
                            @YuriyS The more I see, the happier I am :D
                            $endgroup$
                            – Von Neumann
                            Feb 14 '16 at 20:06






                          • 5




                            $begingroup$
                            Although it adds nothing, I think having $5x^2$ on the left instead of $5$ at the denominator on the right looks even prettier (if possible)
                            $endgroup$
                            – Laurent Duval
                            Feb 15 '16 at 7:14
















                          $begingroup$
                          Great! Another integral that relates two constants! Thank you!
                          $endgroup$
                          – Von Neumann
                          Feb 14 '16 at 19:37




                          $begingroup$
                          Great! Another integral that relates two constants! Thank you!
                          $endgroup$
                          – Von Neumann
                          Feb 14 '16 at 19:37












                          $begingroup$
                          @KimPeek, there is an infinite number of integrals of this kind
                          $endgroup$
                          – Yuriy S
                          Feb 14 '16 at 19:42




                          $begingroup$
                          @KimPeek, there is an infinite number of integrals of this kind
                          $endgroup$
                          – Yuriy S
                          Feb 14 '16 at 19:42




                          4




                          4




                          $begingroup$
                          @YuriyS The more I see, the happier I am :D
                          $endgroup$
                          – Von Neumann
                          Feb 14 '16 at 20:06




                          $begingroup$
                          @YuriyS The more I see, the happier I am :D
                          $endgroup$
                          – Von Neumann
                          Feb 14 '16 at 20:06




                          5




                          5




                          $begingroup$
                          Although it adds nothing, I think having $5x^2$ on the left instead of $5$ at the denominator on the right looks even prettier (if possible)
                          $endgroup$
                          – Laurent Duval
                          Feb 15 '16 at 7:14




                          $begingroup$
                          Although it adds nothing, I think having $5x^2$ on the left instead of $5$ at the denominator on the right looks even prettier (if possible)
                          $endgroup$
                          – Laurent Duval
                          Feb 15 '16 at 7:14











                          20












                          $begingroup$

                          $$int_0^{infty} frac{dx}{(1+x^phi)^phi}=1$$






                          share|cite|improve this answer











                          $endgroup$













                          • $begingroup$
                            Astounding beauty
                            $endgroup$
                            – Von Neumann
                            Apr 24 '16 at 11:04






                          • 1




                            $begingroup$
                            Another integral involving $phi$ that might be surprising at first sight :) $$int_0^inftyfrac1{1+x^2}frac1{1+x^phi}dx=fracpi4.$$
                            $endgroup$
                            – Vladimir Reshetnikov
                            May 27 '16 at 19:27












                          • $begingroup$
                            @VladimirReshetnikov How would you solve both of your integrals one? My mind's blank, and I'm not able to simplify them adequately. :/
                            $endgroup$
                            – Kugelblitz
                            Jun 17 '17 at 4:55










                          • $begingroup$
                            $displaystyleintfrac{dx}{(1+x^phi)^phi}=x , (1+x^phi)^{1-phi}color{gray}{+C}$, that can be checked by differentiation. The second one is really easy. Hint: if you replace $phi$ with $phi^2$, it will still have the same value.
                            $endgroup$
                            – Vladimir Reshetnikov
                            Jun 17 '17 at 21:34


















                          20












                          $begingroup$

                          $$int_0^{infty} frac{dx}{(1+x^phi)^phi}=1$$






                          share|cite|improve this answer











                          $endgroup$













                          • $begingroup$
                            Astounding beauty
                            $endgroup$
                            – Von Neumann
                            Apr 24 '16 at 11:04






                          • 1




                            $begingroup$
                            Another integral involving $phi$ that might be surprising at first sight :) $$int_0^inftyfrac1{1+x^2}frac1{1+x^phi}dx=fracpi4.$$
                            $endgroup$
                            – Vladimir Reshetnikov
                            May 27 '16 at 19:27












                          • $begingroup$
                            @VladimirReshetnikov How would you solve both of your integrals one? My mind's blank, and I'm not able to simplify them adequately. :/
                            $endgroup$
                            – Kugelblitz
                            Jun 17 '17 at 4:55










                          • $begingroup$
                            $displaystyleintfrac{dx}{(1+x^phi)^phi}=x , (1+x^phi)^{1-phi}color{gray}{+C}$, that can be checked by differentiation. The second one is really easy. Hint: if you replace $phi$ with $phi^2$, it will still have the same value.
                            $endgroup$
                            – Vladimir Reshetnikov
                            Jun 17 '17 at 21:34
















                          20












                          20








                          20





                          $begingroup$

                          $$int_0^{infty} frac{dx}{(1+x^phi)^phi}=1$$






                          share|cite|improve this answer











                          $endgroup$



                          $$int_0^{infty} frac{dx}{(1+x^phi)^phi}=1$$







                          share|cite|improve this answer














                          share|cite|improve this answer



                          share|cite|improve this answer








                          edited May 27 '16 at 19:30









                          Vladimir Reshetnikov

                          24.3k4120233




                          24.3k4120233










                          answered Apr 23 '16 at 6:22









                          karvenskarvens

                          3,4971131




                          3,4971131












                          • $begingroup$
                            Astounding beauty
                            $endgroup$
                            – Von Neumann
                            Apr 24 '16 at 11:04






                          • 1




                            $begingroup$
                            Another integral involving $phi$ that might be surprising at first sight :) $$int_0^inftyfrac1{1+x^2}frac1{1+x^phi}dx=fracpi4.$$
                            $endgroup$
                            – Vladimir Reshetnikov
                            May 27 '16 at 19:27












                          • $begingroup$
                            @VladimirReshetnikov How would you solve both of your integrals one? My mind's blank, and I'm not able to simplify them adequately. :/
                            $endgroup$
                            – Kugelblitz
                            Jun 17 '17 at 4:55










                          • $begingroup$
                            $displaystyleintfrac{dx}{(1+x^phi)^phi}=x , (1+x^phi)^{1-phi}color{gray}{+C}$, that can be checked by differentiation. The second one is really easy. Hint: if you replace $phi$ with $phi^2$, it will still have the same value.
                            $endgroup$
                            – Vladimir Reshetnikov
                            Jun 17 '17 at 21:34




















                          • $begingroup$
                            Astounding beauty
                            $endgroup$
                            – Von Neumann
                            Apr 24 '16 at 11:04






                          • 1




                            $begingroup$
                            Another integral involving $phi$ that might be surprising at first sight :) $$int_0^inftyfrac1{1+x^2}frac1{1+x^phi}dx=fracpi4.$$
                            $endgroup$
                            – Vladimir Reshetnikov
                            May 27 '16 at 19:27












                          • $begingroup$
                            @VladimirReshetnikov How would you solve both of your integrals one? My mind's blank, and I'm not able to simplify them adequately. :/
                            $endgroup$
                            – Kugelblitz
                            Jun 17 '17 at 4:55










                          • $begingroup$
                            $displaystyleintfrac{dx}{(1+x^phi)^phi}=x , (1+x^phi)^{1-phi}color{gray}{+C}$, that can be checked by differentiation. The second one is really easy. Hint: if you replace $phi$ with $phi^2$, it will still have the same value.
                            $endgroup$
                            – Vladimir Reshetnikov
                            Jun 17 '17 at 21:34


















                          $begingroup$
                          Astounding beauty
                          $endgroup$
                          – Von Neumann
                          Apr 24 '16 at 11:04




                          $begingroup$
                          Astounding beauty
                          $endgroup$
                          – Von Neumann
                          Apr 24 '16 at 11:04




                          1




                          1




                          $begingroup$
                          Another integral involving $phi$ that might be surprising at first sight :) $$int_0^inftyfrac1{1+x^2}frac1{1+x^phi}dx=fracpi4.$$
                          $endgroup$
                          – Vladimir Reshetnikov
                          May 27 '16 at 19:27






                          $begingroup$
                          Another integral involving $phi$ that might be surprising at first sight :) $$int_0^inftyfrac1{1+x^2}frac1{1+x^phi}dx=fracpi4.$$
                          $endgroup$
                          – Vladimir Reshetnikov
                          May 27 '16 at 19:27














                          $begingroup$
                          @VladimirReshetnikov How would you solve both of your integrals one? My mind's blank, and I'm not able to simplify them adequately. :/
                          $endgroup$
                          – Kugelblitz
                          Jun 17 '17 at 4:55




                          $begingroup$
                          @VladimirReshetnikov How would you solve both of your integrals one? My mind's blank, and I'm not able to simplify them adequately. :/
                          $endgroup$
                          – Kugelblitz
                          Jun 17 '17 at 4:55












                          $begingroup$
                          $displaystyleintfrac{dx}{(1+x^phi)^phi}=x , (1+x^phi)^{1-phi}color{gray}{+C}$, that can be checked by differentiation. The second one is really easy. Hint: if you replace $phi$ with $phi^2$, it will still have the same value.
                          $endgroup$
                          – Vladimir Reshetnikov
                          Jun 17 '17 at 21:34






                          $begingroup$
                          $displaystyleintfrac{dx}{(1+x^phi)^phi}=x , (1+x^phi)^{1-phi}color{gray}{+C}$, that can be checked by differentiation. The second one is really easy. Hint: if you replace $phi$ with $phi^2$, it will still have the same value.
                          $endgroup$
                          – Vladimir Reshetnikov
                          Jun 17 '17 at 21:34













                          19












                          $begingroup$

                          All the following is based on the simple fact that:



                          $$phi=2 cos left( frac{pi}{5} right)=2 sin left( frac{3pi}{10} right)$$



                          These integrals are the small sample of what we can build using this identity:



                          $$frac{1}{2 pi} int_0^{infty} frac{dx}{(1+x)x^{0.7}}=phi-1$$



                          $$frac{1}{1.4 pi} int_0^{infty} frac{dx}{(1+x)^2x^{0.7}}=phi-1$$



                          $$frac{1}{2 pi} int_0^{1} frac{dx}{(1-x)^{0.3}x^{0.7} }=phi-1$$



                          $$frac{5}{3 pi} int_0^{1} frac{x^{0.3}dx}{(1-x)^{0.3} }=phi-1$$



                          $$frac{1}{2 pi} int_1^{infty} frac{dx}{(x-1)^{0.3}x }=phi-1$$



                          $$frac{1}{0.21 pi} int_0^{infty} frac{x^{0.3}dx}{(1+x)^{3} }=phi-1$$



                          Take any tables of definite integrals, find any one that ends in a trig function and set the parameters to obtain $phi$.





                          You can find the following infinite product for $phi$ here



                          $$2 phi=prod_{k=0}^{infty}frac{100k(k+1)+5^2}{100k(k+1)+3^2}$$



                          It's converging slowly, see the link for the proof using the properties of Gamma function.



                          By numerical computation at $50000$ terms this infinite product gives only $5$ correct digits for $phi$, giving $1.618029$ instead of $1.618034$.



                          Using the infinite product for $cos(x)$, we get:



                          $$frac{phi}{2}=prod_{k=1}^{infty}left(1- frac{4}{5^2 (2k-1)^2} right)$$



                          This infinite product at $50000$ terms gives $phi=1.618035$, only $4$ correct digits. This is actually almost the same product, because if we rearrange it we get:



                          $$frac{phi}{2}=prod_{k=0}^{infty}left(frac{100 k (k+1)+21}{100 k (k+1)+25} right)$$



                          I suggest looking at this question for much more interesting product.






                          share|cite|improve this answer











                          $endgroup$


















                            19












                            $begingroup$

                            All the following is based on the simple fact that:



                            $$phi=2 cos left( frac{pi}{5} right)=2 sin left( frac{3pi}{10} right)$$



                            These integrals are the small sample of what we can build using this identity:



                            $$frac{1}{2 pi} int_0^{infty} frac{dx}{(1+x)x^{0.7}}=phi-1$$



                            $$frac{1}{1.4 pi} int_0^{infty} frac{dx}{(1+x)^2x^{0.7}}=phi-1$$



                            $$frac{1}{2 pi} int_0^{1} frac{dx}{(1-x)^{0.3}x^{0.7} }=phi-1$$



                            $$frac{5}{3 pi} int_0^{1} frac{x^{0.3}dx}{(1-x)^{0.3} }=phi-1$$



                            $$frac{1}{2 pi} int_1^{infty} frac{dx}{(x-1)^{0.3}x }=phi-1$$



                            $$frac{1}{0.21 pi} int_0^{infty} frac{x^{0.3}dx}{(1+x)^{3} }=phi-1$$



                            Take any tables of definite integrals, find any one that ends in a trig function and set the parameters to obtain $phi$.





                            You can find the following infinite product for $phi$ here



                            $$2 phi=prod_{k=0}^{infty}frac{100k(k+1)+5^2}{100k(k+1)+3^2}$$



                            It's converging slowly, see the link for the proof using the properties of Gamma function.



                            By numerical computation at $50000$ terms this infinite product gives only $5$ correct digits for $phi$, giving $1.618029$ instead of $1.618034$.



                            Using the infinite product for $cos(x)$, we get:



                            $$frac{phi}{2}=prod_{k=1}^{infty}left(1- frac{4}{5^2 (2k-1)^2} right)$$



                            This infinite product at $50000$ terms gives $phi=1.618035$, only $4$ correct digits. This is actually almost the same product, because if we rearrange it we get:



                            $$frac{phi}{2}=prod_{k=0}^{infty}left(frac{100 k (k+1)+21}{100 k (k+1)+25} right)$$



                            I suggest looking at this question for much more interesting product.






                            share|cite|improve this answer











                            $endgroup$
















                              19












                              19








                              19





                              $begingroup$

                              All the following is based on the simple fact that:



                              $$phi=2 cos left( frac{pi}{5} right)=2 sin left( frac{3pi}{10} right)$$



                              These integrals are the small sample of what we can build using this identity:



                              $$frac{1}{2 pi} int_0^{infty} frac{dx}{(1+x)x^{0.7}}=phi-1$$



                              $$frac{1}{1.4 pi} int_0^{infty} frac{dx}{(1+x)^2x^{0.7}}=phi-1$$



                              $$frac{1}{2 pi} int_0^{1} frac{dx}{(1-x)^{0.3}x^{0.7} }=phi-1$$



                              $$frac{5}{3 pi} int_0^{1} frac{x^{0.3}dx}{(1-x)^{0.3} }=phi-1$$



                              $$frac{1}{2 pi} int_1^{infty} frac{dx}{(x-1)^{0.3}x }=phi-1$$



                              $$frac{1}{0.21 pi} int_0^{infty} frac{x^{0.3}dx}{(1+x)^{3} }=phi-1$$



                              Take any tables of definite integrals, find any one that ends in a trig function and set the parameters to obtain $phi$.





                              You can find the following infinite product for $phi$ here



                              $$2 phi=prod_{k=0}^{infty}frac{100k(k+1)+5^2}{100k(k+1)+3^2}$$



                              It's converging slowly, see the link for the proof using the properties of Gamma function.



                              By numerical computation at $50000$ terms this infinite product gives only $5$ correct digits for $phi$, giving $1.618029$ instead of $1.618034$.



                              Using the infinite product for $cos(x)$, we get:



                              $$frac{phi}{2}=prod_{k=1}^{infty}left(1- frac{4}{5^2 (2k-1)^2} right)$$



                              This infinite product at $50000$ terms gives $phi=1.618035$, only $4$ correct digits. This is actually almost the same product, because if we rearrange it we get:



                              $$frac{phi}{2}=prod_{k=0}^{infty}left(frac{100 k (k+1)+21}{100 k (k+1)+25} right)$$



                              I suggest looking at this question for much more interesting product.






                              share|cite|improve this answer











                              $endgroup$



                              All the following is based on the simple fact that:



                              $$phi=2 cos left( frac{pi}{5} right)=2 sin left( frac{3pi}{10} right)$$



                              These integrals are the small sample of what we can build using this identity:



                              $$frac{1}{2 pi} int_0^{infty} frac{dx}{(1+x)x^{0.7}}=phi-1$$



                              $$frac{1}{1.4 pi} int_0^{infty} frac{dx}{(1+x)^2x^{0.7}}=phi-1$$



                              $$frac{1}{2 pi} int_0^{1} frac{dx}{(1-x)^{0.3}x^{0.7} }=phi-1$$



                              $$frac{5}{3 pi} int_0^{1} frac{x^{0.3}dx}{(1-x)^{0.3} }=phi-1$$



                              $$frac{1}{2 pi} int_1^{infty} frac{dx}{(x-1)^{0.3}x }=phi-1$$



                              $$frac{1}{0.21 pi} int_0^{infty} frac{x^{0.3}dx}{(1+x)^{3} }=phi-1$$



                              Take any tables of definite integrals, find any one that ends in a trig function and set the parameters to obtain $phi$.





                              You can find the following infinite product for $phi$ here



                              $$2 phi=prod_{k=0}^{infty}frac{100k(k+1)+5^2}{100k(k+1)+3^2}$$



                              It's converging slowly, see the link for the proof using the properties of Gamma function.



                              By numerical computation at $50000$ terms this infinite product gives only $5$ correct digits for $phi$, giving $1.618029$ instead of $1.618034$.



                              Using the infinite product for $cos(x)$, we get:



                              $$frac{phi}{2}=prod_{k=1}^{infty}left(1- frac{4}{5^2 (2k-1)^2} right)$$



                              This infinite product at $50000$ terms gives $phi=1.618035$, only $4$ correct digits. This is actually almost the same product, because if we rearrange it we get:



                              $$frac{phi}{2}=prod_{k=0}^{infty}left(frac{100 k (k+1)+21}{100 k (k+1)+25} right)$$



                              I suggest looking at this question for much more interesting product.







                              share|cite|improve this answer














                              share|cite|improve this answer



                              share|cite|improve this answer








                              edited Apr 13 '17 at 12:20









                              Community

                              1




                              1










                              answered Feb 14 '16 at 3:48









                              Yuriy SYuriy S

                              15.8k433117




                              15.8k433117























                                  15












                                  $begingroup$

                                  The length of the logarithmic spiral $rho=e^{2theta}$ up to $theta=0$ is given by



                                  $$int_{-infty}^0sqrt{rho^2+dotrho^2}dtheta=int_{-infty}^0sqrt{1+2^2}e^{2theta}dtheta=phi-frac12.$$






                                  share|cite|improve this answer









                                  $endgroup$













                                  • $begingroup$
                                    Nice. Can you re-adjust the spiral so that length is $phi $ exactly ?
                                    $endgroup$
                                    – Narasimham
                                    Feb 15 '16 at 15:11












                                  • $begingroup$
                                    @Narasimham: I don't see an immediate way to achieve that.
                                    $endgroup$
                                    – Yves Daoust
                                    Feb 15 '16 at 15:22










                                  • $begingroup$
                                    You already have $sqrt{5}$ under your integral. Good example though
                                    $endgroup$
                                    – Yuriy S
                                    Feb 23 '16 at 23:16
















                                  15












                                  $begingroup$

                                  The length of the logarithmic spiral $rho=e^{2theta}$ up to $theta=0$ is given by



                                  $$int_{-infty}^0sqrt{rho^2+dotrho^2}dtheta=int_{-infty}^0sqrt{1+2^2}e^{2theta}dtheta=phi-frac12.$$






                                  share|cite|improve this answer









                                  $endgroup$













                                  • $begingroup$
                                    Nice. Can you re-adjust the spiral so that length is $phi $ exactly ?
                                    $endgroup$
                                    – Narasimham
                                    Feb 15 '16 at 15:11












                                  • $begingroup$
                                    @Narasimham: I don't see an immediate way to achieve that.
                                    $endgroup$
                                    – Yves Daoust
                                    Feb 15 '16 at 15:22










                                  • $begingroup$
                                    You already have $sqrt{5}$ under your integral. Good example though
                                    $endgroup$
                                    – Yuriy S
                                    Feb 23 '16 at 23:16














                                  15












                                  15








                                  15





                                  $begingroup$

                                  The length of the logarithmic spiral $rho=e^{2theta}$ up to $theta=0$ is given by



                                  $$int_{-infty}^0sqrt{rho^2+dotrho^2}dtheta=int_{-infty}^0sqrt{1+2^2}e^{2theta}dtheta=phi-frac12.$$






                                  share|cite|improve this answer









                                  $endgroup$



                                  The length of the logarithmic spiral $rho=e^{2theta}$ up to $theta=0$ is given by



                                  $$int_{-infty}^0sqrt{rho^2+dotrho^2}dtheta=int_{-infty}^0sqrt{1+2^2}e^{2theta}dtheta=phi-frac12.$$







                                  share|cite|improve this answer












                                  share|cite|improve this answer



                                  share|cite|improve this answer










                                  answered Feb 15 '16 at 9:27









                                  Yves DaoustYves Daoust

                                  125k671223




                                  125k671223












                                  • $begingroup$
                                    Nice. Can you re-adjust the spiral so that length is $phi $ exactly ?
                                    $endgroup$
                                    – Narasimham
                                    Feb 15 '16 at 15:11












                                  • $begingroup$
                                    @Narasimham: I don't see an immediate way to achieve that.
                                    $endgroup$
                                    – Yves Daoust
                                    Feb 15 '16 at 15:22










                                  • $begingroup$
                                    You already have $sqrt{5}$ under your integral. Good example though
                                    $endgroup$
                                    – Yuriy S
                                    Feb 23 '16 at 23:16


















                                  • $begingroup$
                                    Nice. Can you re-adjust the spiral so that length is $phi $ exactly ?
                                    $endgroup$
                                    – Narasimham
                                    Feb 15 '16 at 15:11












                                  • $begingroup$
                                    @Narasimham: I don't see an immediate way to achieve that.
                                    $endgroup$
                                    – Yves Daoust
                                    Feb 15 '16 at 15:22










                                  • $begingroup$
                                    You already have $sqrt{5}$ under your integral. Good example though
                                    $endgroup$
                                    – Yuriy S
                                    Feb 23 '16 at 23:16
















                                  $begingroup$
                                  Nice. Can you re-adjust the spiral so that length is $phi $ exactly ?
                                  $endgroup$
                                  – Narasimham
                                  Feb 15 '16 at 15:11






                                  $begingroup$
                                  Nice. Can you re-adjust the spiral so that length is $phi $ exactly ?
                                  $endgroup$
                                  – Narasimham
                                  Feb 15 '16 at 15:11














                                  $begingroup$
                                  @Narasimham: I don't see an immediate way to achieve that.
                                  $endgroup$
                                  – Yves Daoust
                                  Feb 15 '16 at 15:22




                                  $begingroup$
                                  @Narasimham: I don't see an immediate way to achieve that.
                                  $endgroup$
                                  – Yves Daoust
                                  Feb 15 '16 at 15:22












                                  $begingroup$
                                  You already have $sqrt{5}$ under your integral. Good example though
                                  $endgroup$
                                  – Yuriy S
                                  Feb 23 '16 at 23:16




                                  $begingroup$
                                  You already have $sqrt{5}$ under your integral. Good example though
                                  $endgroup$
                                  – Yuriy S
                                  Feb 23 '16 at 23:16











                                  11












                                  $begingroup$

                                  How about this one:




                                  $$int_0^1 frac{dx}{sqrt{x+sqrt{x+sqrt{x+cdots}}}}=frac{2}{phi}-ln phi$$




                                  There is an infinitely nested radical in the denominator.



                                  A finite one is also possible:




                                  $$int_0^{1/16} frac{dx}{sqrt{x+sqrt{x}}}=phi-2ln (phi)-frac12$$







                                  share|cite|improve this answer











                                  $endgroup$













                                  • $begingroup$
                                    The first one is AMAZING!! Thank you for having shared it! :O
                                    $endgroup$
                                    – Von Neumann
                                    Apr 11 '16 at 16:57










                                  • $begingroup$
                                    Might help in the second to note that $ln(phi+1)=2lnphi$
                                    $endgroup$
                                    – πr8
                                    May 3 '16 at 17:36






                                  • 1




                                    $begingroup$
                                    @TimeMaster The 1st integral is only a fancy representation of $$int_0^1 frac{2}{1+sqrt{1+4x}} dx$$
                                    $endgroup$
                                    – Sophie Agnesi
                                    Jun 10 '16 at 7:00










                                  • $begingroup$
                                    @SophieAgnesi, my secret is revealed! Curses!
                                    $endgroup$
                                    – Yuriy S
                                    Jun 10 '16 at 7:55
















                                  11












                                  $begingroup$

                                  How about this one:




                                  $$int_0^1 frac{dx}{sqrt{x+sqrt{x+sqrt{x+cdots}}}}=frac{2}{phi}-ln phi$$




                                  There is an infinitely nested radical in the denominator.



                                  A finite one is also possible:




                                  $$int_0^{1/16} frac{dx}{sqrt{x+sqrt{x}}}=phi-2ln (phi)-frac12$$







                                  share|cite|improve this answer











                                  $endgroup$













                                  • $begingroup$
                                    The first one is AMAZING!! Thank you for having shared it! :O
                                    $endgroup$
                                    – Von Neumann
                                    Apr 11 '16 at 16:57










                                  • $begingroup$
                                    Might help in the second to note that $ln(phi+1)=2lnphi$
                                    $endgroup$
                                    – πr8
                                    May 3 '16 at 17:36






                                  • 1




                                    $begingroup$
                                    @TimeMaster The 1st integral is only a fancy representation of $$int_0^1 frac{2}{1+sqrt{1+4x}} dx$$
                                    $endgroup$
                                    – Sophie Agnesi
                                    Jun 10 '16 at 7:00










                                  • $begingroup$
                                    @SophieAgnesi, my secret is revealed! Curses!
                                    $endgroup$
                                    – Yuriy S
                                    Jun 10 '16 at 7:55














                                  11












                                  11








                                  11





                                  $begingroup$

                                  How about this one:




                                  $$int_0^1 frac{dx}{sqrt{x+sqrt{x+sqrt{x+cdots}}}}=frac{2}{phi}-ln phi$$




                                  There is an infinitely nested radical in the denominator.



                                  A finite one is also possible:




                                  $$int_0^{1/16} frac{dx}{sqrt{x+sqrt{x}}}=phi-2ln (phi)-frac12$$







                                  share|cite|improve this answer











                                  $endgroup$



                                  How about this one:




                                  $$int_0^1 frac{dx}{sqrt{x+sqrt{x+sqrt{x+cdots}}}}=frac{2}{phi}-ln phi$$




                                  There is an infinitely nested radical in the denominator.



                                  A finite one is also possible:




                                  $$int_0^{1/16} frac{dx}{sqrt{x+sqrt{x}}}=phi-2ln (phi)-frac12$$








                                  share|cite|improve this answer














                                  share|cite|improve this answer



                                  share|cite|improve this answer








                                  edited Jun 13 '16 at 18:54

























                                  answered Apr 11 '16 at 12:47









                                  Yuriy SYuriy S

                                  15.8k433117




                                  15.8k433117












                                  • $begingroup$
                                    The first one is AMAZING!! Thank you for having shared it! :O
                                    $endgroup$
                                    – Von Neumann
                                    Apr 11 '16 at 16:57










                                  • $begingroup$
                                    Might help in the second to note that $ln(phi+1)=2lnphi$
                                    $endgroup$
                                    – πr8
                                    May 3 '16 at 17:36






                                  • 1




                                    $begingroup$
                                    @TimeMaster The 1st integral is only a fancy representation of $$int_0^1 frac{2}{1+sqrt{1+4x}} dx$$
                                    $endgroup$
                                    – Sophie Agnesi
                                    Jun 10 '16 at 7:00










                                  • $begingroup$
                                    @SophieAgnesi, my secret is revealed! Curses!
                                    $endgroup$
                                    – Yuriy S
                                    Jun 10 '16 at 7:55


















                                  • $begingroup$
                                    The first one is AMAZING!! Thank you for having shared it! :O
                                    $endgroup$
                                    – Von Neumann
                                    Apr 11 '16 at 16:57










                                  • $begingroup$
                                    Might help in the second to note that $ln(phi+1)=2lnphi$
                                    $endgroup$
                                    – πr8
                                    May 3 '16 at 17:36






                                  • 1




                                    $begingroup$
                                    @TimeMaster The 1st integral is only a fancy representation of $$int_0^1 frac{2}{1+sqrt{1+4x}} dx$$
                                    $endgroup$
                                    – Sophie Agnesi
                                    Jun 10 '16 at 7:00










                                  • $begingroup$
                                    @SophieAgnesi, my secret is revealed! Curses!
                                    $endgroup$
                                    – Yuriy S
                                    Jun 10 '16 at 7:55
















                                  $begingroup$
                                  The first one is AMAZING!! Thank you for having shared it! :O
                                  $endgroup$
                                  – Von Neumann
                                  Apr 11 '16 at 16:57




                                  $begingroup$
                                  The first one is AMAZING!! Thank you for having shared it! :O
                                  $endgroup$
                                  – Von Neumann
                                  Apr 11 '16 at 16:57












                                  $begingroup$
                                  Might help in the second to note that $ln(phi+1)=2lnphi$
                                  $endgroup$
                                  – πr8
                                  May 3 '16 at 17:36




                                  $begingroup$
                                  Might help in the second to note that $ln(phi+1)=2lnphi$
                                  $endgroup$
                                  – πr8
                                  May 3 '16 at 17:36




                                  1




                                  1




                                  $begingroup$
                                  @TimeMaster The 1st integral is only a fancy representation of $$int_0^1 frac{2}{1+sqrt{1+4x}} dx$$
                                  $endgroup$
                                  – Sophie Agnesi
                                  Jun 10 '16 at 7:00




                                  $begingroup$
                                  @TimeMaster The 1st integral is only a fancy representation of $$int_0^1 frac{2}{1+sqrt{1+4x}} dx$$
                                  $endgroup$
                                  – Sophie Agnesi
                                  Jun 10 '16 at 7:00












                                  $begingroup$
                                  @SophieAgnesi, my secret is revealed! Curses!
                                  $endgroup$
                                  – Yuriy S
                                  Jun 10 '16 at 7:55




                                  $begingroup$
                                  @SophieAgnesi, my secret is revealed! Curses!
                                  $endgroup$
                                  – Yuriy S
                                  Jun 10 '16 at 7:55











                                  9












                                  $begingroup$

                                  $$int_0^infty x(2x-1),delta(x^2-x-1),dx$$





                                  Update:



                                  As pointed by Yuriy, we must take into account the derivative of the argument of the $delta$ function. This is why the corrective factor $2x-1$ appears.



                                  More generally,



                                  $$int_I x|g'(x)|delta(g(x)),dx$$ evaluates to the root of $g$ contained in the interval $I$, provided there is only one. The first factor $x$ can be replaced by any function $f(x)$ to yield the value of that function at the root.






                                  share|cite|improve this answer











                                  $endgroup$













                                  • $begingroup$
                                    Beautiful!! Dirac Delta. Very easy and elegant, thank you!
                                    $endgroup$
                                    – Von Neumann
                                    Feb 14 '16 at 17:43










                                  • $begingroup$
                                    A great idea, actually! We can do it for any algebraic number, it seems
                                    $endgroup$
                                    – Yuriy S
                                    Feb 14 '16 at 19:24










                                  • $begingroup$
                                    @YuriyS: yep, provided you isolate the desired root in an interval.
                                    $endgroup$
                                    – Yves Daoust
                                    Feb 14 '16 at 19:36










                                  • $begingroup$
                                    Actually, Wolframalpha gives another value for this integral: wolframalpha.com/input/…
                                    $endgroup$
                                    – Yuriy S
                                    Feb 14 '16 at 20:19










                                  • $begingroup$
                                    In general $delta [g(x)]=sum_k frac{delta (x-x_k)}{| g'(x_k)|}$
                                    $endgroup$
                                    – Yuriy S
                                    Feb 14 '16 at 20:25
















                                  9












                                  $begingroup$

                                  $$int_0^infty x(2x-1),delta(x^2-x-1),dx$$





                                  Update:



                                  As pointed by Yuriy, we must take into account the derivative of the argument of the $delta$ function. This is why the corrective factor $2x-1$ appears.



                                  More generally,



                                  $$int_I x|g'(x)|delta(g(x)),dx$$ evaluates to the root of $g$ contained in the interval $I$, provided there is only one. The first factor $x$ can be replaced by any function $f(x)$ to yield the value of that function at the root.






                                  share|cite|improve this answer











                                  $endgroup$













                                  • $begingroup$
                                    Beautiful!! Dirac Delta. Very easy and elegant, thank you!
                                    $endgroup$
                                    – Von Neumann
                                    Feb 14 '16 at 17:43










                                  • $begingroup$
                                    A great idea, actually! We can do it for any algebraic number, it seems
                                    $endgroup$
                                    – Yuriy S
                                    Feb 14 '16 at 19:24










                                  • $begingroup$
                                    @YuriyS: yep, provided you isolate the desired root in an interval.
                                    $endgroup$
                                    – Yves Daoust
                                    Feb 14 '16 at 19:36










                                  • $begingroup$
                                    Actually, Wolframalpha gives another value for this integral: wolframalpha.com/input/…
                                    $endgroup$
                                    – Yuriy S
                                    Feb 14 '16 at 20:19










                                  • $begingroup$
                                    In general $delta [g(x)]=sum_k frac{delta (x-x_k)}{| g'(x_k)|}$
                                    $endgroup$
                                    – Yuriy S
                                    Feb 14 '16 at 20:25














                                  9












                                  9








                                  9





                                  $begingroup$

                                  $$int_0^infty x(2x-1),delta(x^2-x-1),dx$$





                                  Update:



                                  As pointed by Yuriy, we must take into account the derivative of the argument of the $delta$ function. This is why the corrective factor $2x-1$ appears.



                                  More generally,



                                  $$int_I x|g'(x)|delta(g(x)),dx$$ evaluates to the root of $g$ contained in the interval $I$, provided there is only one. The first factor $x$ can be replaced by any function $f(x)$ to yield the value of that function at the root.






                                  share|cite|improve this answer











                                  $endgroup$



                                  $$int_0^infty x(2x-1),delta(x^2-x-1),dx$$





                                  Update:



                                  As pointed by Yuriy, we must take into account the derivative of the argument of the $delta$ function. This is why the corrective factor $2x-1$ appears.



                                  More generally,



                                  $$int_I x|g'(x)|delta(g(x)),dx$$ evaluates to the root of $g$ contained in the interval $I$, provided there is only one. The first factor $x$ can be replaced by any function $f(x)$ to yield the value of that function at the root.







                                  share|cite|improve this answer














                                  share|cite|improve this answer



                                  share|cite|improve this answer








                                  edited Feb 14 '16 at 21:17

























                                  answered Feb 14 '16 at 16:55









                                  Yves DaoustYves Daoust

                                  125k671223




                                  125k671223












                                  • $begingroup$
                                    Beautiful!! Dirac Delta. Very easy and elegant, thank you!
                                    $endgroup$
                                    – Von Neumann
                                    Feb 14 '16 at 17:43










                                  • $begingroup$
                                    A great idea, actually! We can do it for any algebraic number, it seems
                                    $endgroup$
                                    – Yuriy S
                                    Feb 14 '16 at 19:24










                                  • $begingroup$
                                    @YuriyS: yep, provided you isolate the desired root in an interval.
                                    $endgroup$
                                    – Yves Daoust
                                    Feb 14 '16 at 19:36










                                  • $begingroup$
                                    Actually, Wolframalpha gives another value for this integral: wolframalpha.com/input/…
                                    $endgroup$
                                    – Yuriy S
                                    Feb 14 '16 at 20:19










                                  • $begingroup$
                                    In general $delta [g(x)]=sum_k frac{delta (x-x_k)}{| g'(x_k)|}$
                                    $endgroup$
                                    – Yuriy S
                                    Feb 14 '16 at 20:25


















                                  • $begingroup$
                                    Beautiful!! Dirac Delta. Very easy and elegant, thank you!
                                    $endgroup$
                                    – Von Neumann
                                    Feb 14 '16 at 17:43










                                  • $begingroup$
                                    A great idea, actually! We can do it for any algebraic number, it seems
                                    $endgroup$
                                    – Yuriy S
                                    Feb 14 '16 at 19:24










                                  • $begingroup$
                                    @YuriyS: yep, provided you isolate the desired root in an interval.
                                    $endgroup$
                                    – Yves Daoust
                                    Feb 14 '16 at 19:36










                                  • $begingroup$
                                    Actually, Wolframalpha gives another value for this integral: wolframalpha.com/input/…
                                    $endgroup$
                                    – Yuriy S
                                    Feb 14 '16 at 20:19










                                  • $begingroup$
                                    In general $delta [g(x)]=sum_k frac{delta (x-x_k)}{| g'(x_k)|}$
                                    $endgroup$
                                    – Yuriy S
                                    Feb 14 '16 at 20:25
















                                  $begingroup$
                                  Beautiful!! Dirac Delta. Very easy and elegant, thank you!
                                  $endgroup$
                                  – Von Neumann
                                  Feb 14 '16 at 17:43




                                  $begingroup$
                                  Beautiful!! Dirac Delta. Very easy and elegant, thank you!
                                  $endgroup$
                                  – Von Neumann
                                  Feb 14 '16 at 17:43












                                  $begingroup$
                                  A great idea, actually! We can do it for any algebraic number, it seems
                                  $endgroup$
                                  – Yuriy S
                                  Feb 14 '16 at 19:24




                                  $begingroup$
                                  A great idea, actually! We can do it for any algebraic number, it seems
                                  $endgroup$
                                  – Yuriy S
                                  Feb 14 '16 at 19:24












                                  $begingroup$
                                  @YuriyS: yep, provided you isolate the desired root in an interval.
                                  $endgroup$
                                  – Yves Daoust
                                  Feb 14 '16 at 19:36




                                  $begingroup$
                                  @YuriyS: yep, provided you isolate the desired root in an interval.
                                  $endgroup$
                                  – Yves Daoust
                                  Feb 14 '16 at 19:36












                                  $begingroup$
                                  Actually, Wolframalpha gives another value for this integral: wolframalpha.com/input/…
                                  $endgroup$
                                  – Yuriy S
                                  Feb 14 '16 at 20:19




                                  $begingroup$
                                  Actually, Wolframalpha gives another value for this integral: wolframalpha.com/input/…
                                  $endgroup$
                                  – Yuriy S
                                  Feb 14 '16 at 20:19












                                  $begingroup$
                                  In general $delta [g(x)]=sum_k frac{delta (x-x_k)}{| g'(x_k)|}$
                                  $endgroup$
                                  – Yuriy S
                                  Feb 14 '16 at 20:25




                                  $begingroup$
                                  In general $delta [g(x)]=sum_k frac{delta (x-x_k)}{| g'(x_k)|}$
                                  $endgroup$
                                  – Yuriy S
                                  Feb 14 '16 at 20:25











                                  7












                                  $begingroup$

                                  I am not taking credit for this. I am just posting this because it answers the question. I give Felix Marin and Olivier Oloa complete credit for these results.



                                  $$int_0^{pi/2} ln(1+4sin^2 x)text{ d}x=pilogleft(varphiright)$$



                                  and



                                  $$int_0^{pi/2} ln(1+4sin^4 x)text{ d}x=pilog frac{varphi+sqrt{varphi}}{2}$$



                                  Again, not mine. But they definitely deserve to be here






                                  share|cite|improve this answer









                                  $endgroup$













                                  • $begingroup$
                                    Beautiful! Thank you for having posted them here. The first one is so beautiful!!
                                    $endgroup$
                                    – Von Neumann
                                    Apr 2 '16 at 13:42
















                                  7












                                  $begingroup$

                                  I am not taking credit for this. I am just posting this because it answers the question. I give Felix Marin and Olivier Oloa complete credit for these results.



                                  $$int_0^{pi/2} ln(1+4sin^2 x)text{ d}x=pilogleft(varphiright)$$



                                  and



                                  $$int_0^{pi/2} ln(1+4sin^4 x)text{ d}x=pilog frac{varphi+sqrt{varphi}}{2}$$



                                  Again, not mine. But they definitely deserve to be here






                                  share|cite|improve this answer









                                  $endgroup$













                                  • $begingroup$
                                    Beautiful! Thank you for having posted them here. The first one is so beautiful!!
                                    $endgroup$
                                    – Von Neumann
                                    Apr 2 '16 at 13:42














                                  7












                                  7








                                  7





                                  $begingroup$

                                  I am not taking credit for this. I am just posting this because it answers the question. I give Felix Marin and Olivier Oloa complete credit for these results.



                                  $$int_0^{pi/2} ln(1+4sin^2 x)text{ d}x=pilogleft(varphiright)$$



                                  and



                                  $$int_0^{pi/2} ln(1+4sin^4 x)text{ d}x=pilog frac{varphi+sqrt{varphi}}{2}$$



                                  Again, not mine. But they definitely deserve to be here






                                  share|cite|improve this answer









                                  $endgroup$



                                  I am not taking credit for this. I am just posting this because it answers the question. I give Felix Marin and Olivier Oloa complete credit for these results.



                                  $$int_0^{pi/2} ln(1+4sin^2 x)text{ d}x=pilogleft(varphiright)$$



                                  and



                                  $$int_0^{pi/2} ln(1+4sin^4 x)text{ d}x=pilog frac{varphi+sqrt{varphi}}{2}$$



                                  Again, not mine. But they definitely deserve to be here







                                  share|cite|improve this answer












                                  share|cite|improve this answer



                                  share|cite|improve this answer










                                  answered Apr 2 '16 at 4:57







                                  user311151



















                                  • $begingroup$
                                    Beautiful! Thank you for having posted them here. The first one is so beautiful!!
                                    $endgroup$
                                    – Von Neumann
                                    Apr 2 '16 at 13:42


















                                  • $begingroup$
                                    Beautiful! Thank you for having posted them here. The first one is so beautiful!!
                                    $endgroup$
                                    – Von Neumann
                                    Apr 2 '16 at 13:42
















                                  $begingroup$
                                  Beautiful! Thank you for having posted them here. The first one is so beautiful!!
                                  $endgroup$
                                  – Von Neumann
                                  Apr 2 '16 at 13:42




                                  $begingroup$
                                  Beautiful! Thank you for having posted them here. The first one is so beautiful!!
                                  $endgroup$
                                  – Von Neumann
                                  Apr 2 '16 at 13:42











                                  5












                                  $begingroup$

                                  $$
                                  int_0^1 frac{1+x^8}{1+x^{10}}dx=frac{pi}{phi^5-8}
                                  $$






                                  share|cite|improve this answer









                                  $endgroup$


















                                    5












                                    $begingroup$

                                    $$
                                    int_0^1 frac{1+x^8}{1+x^{10}}dx=frac{pi}{phi^5-8}
                                    $$






                                    share|cite|improve this answer









                                    $endgroup$
















                                      5












                                      5








                                      5





                                      $begingroup$

                                      $$
                                      int_0^1 frac{1+x^8}{1+x^{10}}dx=frac{pi}{phi^5-8}
                                      $$






                                      share|cite|improve this answer









                                      $endgroup$



                                      $$
                                      int_0^1 frac{1+x^8}{1+x^{10}}dx=frac{pi}{phi^5-8}
                                      $$







                                      share|cite|improve this answer












                                      share|cite|improve this answer



                                      share|cite|improve this answer










                                      answered Apr 28 '16 at 20:52







                                      user334593






























                                          5












                                          $begingroup$

                                          Consider the sequence



                                          $1,2,2,3,3,4,4,4,...$



                                          where $a_1=1,a_{n+1}in{a_n,a_n+1}$, and $a_n$ is the number of times $n$ occurs in the sequence. Then if we assume that $a_n$ grows asymptotically as $alpha n^beta$, we get



                                          $alpha=phi^{1/{phi^2}}$



                                          $beta=1/phi$.



                                          I saw this is a textbook problem on asymptotic analysis. It turns out that for all $n$ the asymptotic expression is well within one unit of the actual $a_n$.






                                          share|cite|improve this answer











                                          $endgroup$













                                          • $begingroup$
                                            I give up! How do I put braces around an explicitly written set!?!
                                            $endgroup$
                                            – Oscar Lanzi
                                            Apr 29 '16 at 10:46






                                          • 1




                                            $begingroup$
                                            Use { and } instead of the normal braces.
                                            $endgroup$
                                            – Marra
                                            May 2 '16 at 13:57
















                                          5












                                          $begingroup$

                                          Consider the sequence



                                          $1,2,2,3,3,4,4,4,...$



                                          where $a_1=1,a_{n+1}in{a_n,a_n+1}$, and $a_n$ is the number of times $n$ occurs in the sequence. Then if we assume that $a_n$ grows asymptotically as $alpha n^beta$, we get



                                          $alpha=phi^{1/{phi^2}}$



                                          $beta=1/phi$.



                                          I saw this is a textbook problem on asymptotic analysis. It turns out that for all $n$ the asymptotic expression is well within one unit of the actual $a_n$.






                                          share|cite|improve this answer











                                          $endgroup$













                                          • $begingroup$
                                            I give up! How do I put braces around an explicitly written set!?!
                                            $endgroup$
                                            – Oscar Lanzi
                                            Apr 29 '16 at 10:46






                                          • 1




                                            $begingroup$
                                            Use { and } instead of the normal braces.
                                            $endgroup$
                                            – Marra
                                            May 2 '16 at 13:57














                                          5












                                          5








                                          5





                                          $begingroup$

                                          Consider the sequence



                                          $1,2,2,3,3,4,4,4,...$



                                          where $a_1=1,a_{n+1}in{a_n,a_n+1}$, and $a_n$ is the number of times $n$ occurs in the sequence. Then if we assume that $a_n$ grows asymptotically as $alpha n^beta$, we get



                                          $alpha=phi^{1/{phi^2}}$



                                          $beta=1/phi$.



                                          I saw this is a textbook problem on asymptotic analysis. It turns out that for all $n$ the asymptotic expression is well within one unit of the actual $a_n$.






                                          share|cite|improve this answer











                                          $endgroup$



                                          Consider the sequence



                                          $1,2,2,3,3,4,4,4,...$



                                          where $a_1=1,a_{n+1}in{a_n,a_n+1}$, and $a_n$ is the number of times $n$ occurs in the sequence. Then if we assume that $a_n$ grows asymptotically as $alpha n^beta$, we get



                                          $alpha=phi^{1/{phi^2}}$



                                          $beta=1/phi$.



                                          I saw this is a textbook problem on asymptotic analysis. It turns out that for all $n$ the asymptotic expression is well within one unit of the actual $a_n$.







                                          share|cite|improve this answer














                                          share|cite|improve this answer



                                          share|cite|improve this answer








                                          edited May 2 '16 at 14:25

























                                          answered Apr 29 '16 at 10:35









                                          Oscar LanziOscar Lanzi

                                          12.4k12036




                                          12.4k12036












                                          • $begingroup$
                                            I give up! How do I put braces around an explicitly written set!?!
                                            $endgroup$
                                            – Oscar Lanzi
                                            Apr 29 '16 at 10:46






                                          • 1




                                            $begingroup$
                                            Use { and } instead of the normal braces.
                                            $endgroup$
                                            – Marra
                                            May 2 '16 at 13:57


















                                          • $begingroup$
                                            I give up! How do I put braces around an explicitly written set!?!
                                            $endgroup$
                                            – Oscar Lanzi
                                            Apr 29 '16 at 10:46






                                          • 1




                                            $begingroup$
                                            Use { and } instead of the normal braces.
                                            $endgroup$
                                            – Marra
                                            May 2 '16 at 13:57
















                                          $begingroup$
                                          I give up! How do I put braces around an explicitly written set!?!
                                          $endgroup$
                                          – Oscar Lanzi
                                          Apr 29 '16 at 10:46




                                          $begingroup$
                                          I give up! How do I put braces around an explicitly written set!?!
                                          $endgroup$
                                          – Oscar Lanzi
                                          Apr 29 '16 at 10:46




                                          1




                                          1




                                          $begingroup$
                                          Use { and } instead of the normal braces.
                                          $endgroup$
                                          – Marra
                                          May 2 '16 at 13:57




                                          $begingroup$
                                          Use { and } instead of the normal braces.
                                          $endgroup$
                                          – Marra
                                          May 2 '16 at 13:57











                                          4












                                          $begingroup$

                                          So you said that series are OK, so I will offer a few:



                                          $$phi=frac{13}{8}+sum_{n=0}^infty frac{(-1)^{n+1}(2n+1)!}{n!(n+1)!4^{(2n+3)}}$$



                                          $$phi=2cos (pi/5)=2sum_{k=0}^infty frac{((-1)^k (pi/5)^{2 k}}{(2k)!}$$



                                          $$phi=frac{1}{2}+frac{sqrt{5}}{2}=frac{1}{2}+sum_{n=0}^infty 4^{-n}binom{1/2}{n}$$






                                          share|cite|improve this answer









                                          $endgroup$


















                                            4












                                            $begingroup$

                                            So you said that series are OK, so I will offer a few:



                                            $$phi=frac{13}{8}+sum_{n=0}^infty frac{(-1)^{n+1}(2n+1)!}{n!(n+1)!4^{(2n+3)}}$$



                                            $$phi=2cos (pi/5)=2sum_{k=0}^infty frac{((-1)^k (pi/5)^{2 k}}{(2k)!}$$



                                            $$phi=frac{1}{2}+frac{sqrt{5}}{2}=frac{1}{2}+sum_{n=0}^infty 4^{-n}binom{1/2}{n}$$






                                            share|cite|improve this answer









                                            $endgroup$
















                                              4












                                              4








                                              4





                                              $begingroup$

                                              So you said that series are OK, so I will offer a few:



                                              $$phi=frac{13}{8}+sum_{n=0}^infty frac{(-1)^{n+1}(2n+1)!}{n!(n+1)!4^{(2n+3)}}$$



                                              $$phi=2cos (pi/5)=2sum_{k=0}^infty frac{((-1)^k (pi/5)^{2 k}}{(2k)!}$$



                                              $$phi=frac{1}{2}+frac{sqrt{5}}{2}=frac{1}{2}+sum_{n=0}^infty 4^{-n}binom{1/2}{n}$$






                                              share|cite|improve this answer









                                              $endgroup$



                                              So you said that series are OK, so I will offer a few:



                                              $$phi=frac{13}{8}+sum_{n=0}^infty frac{(-1)^{n+1}(2n+1)!}{n!(n+1)!4^{(2n+3)}}$$



                                              $$phi=2cos (pi/5)=2sum_{k=0}^infty frac{((-1)^k (pi/5)^{2 k}}{(2k)!}$$



                                              $$phi=frac{1}{2}+frac{sqrt{5}}{2}=frac{1}{2}+sum_{n=0}^infty 4^{-n}binom{1/2}{n}$$







                                              share|cite|improve this answer












                                              share|cite|improve this answer



                                              share|cite|improve this answer










                                              answered Feb 20 '16 at 2:41







                                              user311151






























                                                  4












                                                  $begingroup$

                                                  $$
                                                  int_0^infty frac{1}{1+x^{10}}dx=frac{phipi}{5}
                                                  $$






                                                  share|cite|improve this answer









                                                  $endgroup$













                                                  • $begingroup$
                                                    Awesome one!!!!
                                                    $endgroup$
                                                    – Von Neumann
                                                    May 4 '16 at 9:27
















                                                  4












                                                  $begingroup$

                                                  $$
                                                  int_0^infty frac{1}{1+x^{10}}dx=frac{phipi}{5}
                                                  $$






                                                  share|cite|improve this answer









                                                  $endgroup$













                                                  • $begingroup$
                                                    Awesome one!!!!
                                                    $endgroup$
                                                    – Von Neumann
                                                    May 4 '16 at 9:27














                                                  4












                                                  4








                                                  4





                                                  $begingroup$

                                                  $$
                                                  int_0^infty frac{1}{1+x^{10}}dx=frac{phipi}{5}
                                                  $$






                                                  share|cite|improve this answer









                                                  $endgroup$



                                                  $$
                                                  int_0^infty frac{1}{1+x^{10}}dx=frac{phipi}{5}
                                                  $$







                                                  share|cite|improve this answer












                                                  share|cite|improve this answer



                                                  share|cite|improve this answer










                                                  answered Apr 29 '16 at 19:50







                                                  user334593



















                                                  • $begingroup$
                                                    Awesome one!!!!
                                                    $endgroup$
                                                    – Von Neumann
                                                    May 4 '16 at 9:27


















                                                  • $begingroup$
                                                    Awesome one!!!!
                                                    $endgroup$
                                                    – Von Neumann
                                                    May 4 '16 at 9:27
















                                                  $begingroup$
                                                  Awesome one!!!!
                                                  $endgroup$
                                                  – Von Neumann
                                                  May 4 '16 at 9:27




                                                  $begingroup$
                                                  Awesome one!!!!
                                                  $endgroup$
                                                  – Von Neumann
                                                  May 4 '16 at 9:27











                                                  3












                                                  $begingroup$

                                                  -I remember really liking this one:



                                                  $$int_0^1 int_0^1 frac{text{dx dy}}{varphi^6-x^2y^2}=frac{pi^2-18log^2varphi}{24varphi^3}$$



                                                  I most liked it because it was specific to $varphi$



                                                  -Also, we can note this M.SE result (with some interpolation)



                                                  $$int_0^1 frac{log (1+x^{alpha+sqrt{alpha^2-1}})}{1+x}text{dx}=$$$$frac{pi^2}{12}left(frac{alpha}{2}+sqrt{alpha^2-1}right)+log(varphi)log(2)log(sqrt{alpha+1}+sqrt{alpha-1})log(text{something})$$



                                                  Perhaps someone can help me fill in $text{"something"}$






                                                  share|cite|improve this answer











                                                  $endgroup$


















                                                    3












                                                    $begingroup$

                                                    -I remember really liking this one:



                                                    $$int_0^1 int_0^1 frac{text{dx dy}}{varphi^6-x^2y^2}=frac{pi^2-18log^2varphi}{24varphi^3}$$



                                                    I most liked it because it was specific to $varphi$



                                                    -Also, we can note this M.SE result (with some interpolation)



                                                    $$int_0^1 frac{log (1+x^{alpha+sqrt{alpha^2-1}})}{1+x}text{dx}=$$$$frac{pi^2}{12}left(frac{alpha}{2}+sqrt{alpha^2-1}right)+log(varphi)log(2)log(sqrt{alpha+1}+sqrt{alpha-1})log(text{something})$$



                                                    Perhaps someone can help me fill in $text{"something"}$






                                                    share|cite|improve this answer











                                                    $endgroup$
















                                                      3












                                                      3








                                                      3





                                                      $begingroup$

                                                      -I remember really liking this one:



                                                      $$int_0^1 int_0^1 frac{text{dx dy}}{varphi^6-x^2y^2}=frac{pi^2-18log^2varphi}{24varphi^3}$$



                                                      I most liked it because it was specific to $varphi$



                                                      -Also, we can note this M.SE result (with some interpolation)



                                                      $$int_0^1 frac{log (1+x^{alpha+sqrt{alpha^2-1}})}{1+x}text{dx}=$$$$frac{pi^2}{12}left(frac{alpha}{2}+sqrt{alpha^2-1}right)+log(varphi)log(2)log(sqrt{alpha+1}+sqrt{alpha-1})log(text{something})$$



                                                      Perhaps someone can help me fill in $text{"something"}$






                                                      share|cite|improve this answer











                                                      $endgroup$



                                                      -I remember really liking this one:



                                                      $$int_0^1 int_0^1 frac{text{dx dy}}{varphi^6-x^2y^2}=frac{pi^2-18log^2varphi}{24varphi^3}$$



                                                      I most liked it because it was specific to $varphi$



                                                      -Also, we can note this M.SE result (with some interpolation)



                                                      $$int_0^1 frac{log (1+x^{alpha+sqrt{alpha^2-1}})}{1+x}text{dx}=$$$$frac{pi^2}{12}left(frac{alpha}{2}+sqrt{alpha^2-1}right)+log(varphi)log(2)log(sqrt{alpha+1}+sqrt{alpha-1})log(text{something})$$



                                                      Perhaps someone can help me fill in $text{"something"}$







                                                      share|cite|improve this answer














                                                      share|cite|improve this answer



                                                      share|cite|improve this answer








                                                      edited Apr 13 '17 at 12:21









                                                      Community

                                                      1




                                                      1










                                                      answered Apr 23 '16 at 6:11







                                                      user331275






























                                                          3












                                                          $begingroup$

                                                          Here is another one
                                                          $$
                                                          int_0^infty frac{1}{5^{frac{x}{4}}+5^{frac{1}{2}}-5^0}dx=phi
                                                          $$






                                                          share|cite|improve this answer









                                                          $endgroup$


















                                                            3












                                                            $begingroup$

                                                            Here is another one
                                                            $$
                                                            int_0^infty frac{1}{5^{frac{x}{4}}+5^{frac{1}{2}}-5^0}dx=phi
                                                            $$






                                                            share|cite|improve this answer









                                                            $endgroup$
















                                                              3












                                                              3








                                                              3





                                                              $begingroup$

                                                              Here is another one
                                                              $$
                                                              int_0^infty frac{1}{5^{frac{x}{4}}+5^{frac{1}{2}}-5^0}dx=phi
                                                              $$






                                                              share|cite|improve this answer









                                                              $endgroup$



                                                              Here is another one
                                                              $$
                                                              int_0^infty frac{1}{5^{frac{x}{4}}+5^{frac{1}{2}}-5^0}dx=phi
                                                              $$







                                                              share|cite|improve this answer












                                                              share|cite|improve this answer



                                                              share|cite|improve this answer










                                                              answered Apr 29 '16 at 5:24







                                                              user334593






























                                                                  3












                                                                  $begingroup$

                                                                  This one is a bit messy.



                                                                  $$
                                                                  int_0^infty frac{1}{(sqrt5^x)^{2^{-(sqrt5-1)}}+sqrt5-1}dx=2^{phi^{-3}}cdotphi
                                                                  $$






                                                                  share|cite|improve this answer









                                                                  $endgroup$


















                                                                    3












                                                                    $begingroup$

                                                                    This one is a bit messy.



                                                                    $$
                                                                    int_0^infty frac{1}{(sqrt5^x)^{2^{-(sqrt5-1)}}+sqrt5-1}dx=2^{phi^{-3}}cdotphi
                                                                    $$






                                                                    share|cite|improve this answer









                                                                    $endgroup$
















                                                                      3












                                                                      3








                                                                      3





                                                                      $begingroup$

                                                                      This one is a bit messy.



                                                                      $$
                                                                      int_0^infty frac{1}{(sqrt5^x)^{2^{-(sqrt5-1)}}+sqrt5-1}dx=2^{phi^{-3}}cdotphi
                                                                      $$






                                                                      share|cite|improve this answer









                                                                      $endgroup$



                                                                      This one is a bit messy.



                                                                      $$
                                                                      int_0^infty frac{1}{(sqrt5^x)^{2^{-(sqrt5-1)}}+sqrt5-1}dx=2^{phi^{-3}}cdotphi
                                                                      $$







                                                                      share|cite|improve this answer












                                                                      share|cite|improve this answer



                                                                      share|cite|improve this answer










                                                                      answered Apr 29 '16 at 17:01







                                                                      user334593






























                                                                          3












                                                                          $begingroup$

                                                                          $$int_0^infty frac{1}{1+x^{frac{10}{3}}}dx=frac{3pi}{5phi}$$






                                                                          share|cite|improve this answer









                                                                          $endgroup$


















                                                                            3












                                                                            $begingroup$

                                                                            $$int_0^infty frac{1}{1+x^{frac{10}{3}}}dx=frac{3pi}{5phi}$$






                                                                            share|cite|improve this answer









                                                                            $endgroup$
















                                                                              3












                                                                              3








                                                                              3





                                                                              $begingroup$

                                                                              $$int_0^infty frac{1}{1+x^{frac{10}{3}}}dx=frac{3pi}{5phi}$$






                                                                              share|cite|improve this answer









                                                                              $endgroup$



                                                                              $$int_0^infty frac{1}{1+x^{frac{10}{3}}}dx=frac{3pi}{5phi}$$







                                                                              share|cite|improve this answer












                                                                              share|cite|improve this answer



                                                                              share|cite|improve this answer










                                                                              answered May 2 '16 at 13:11







                                                                              user335850






























                                                                                  3












                                                                                  $begingroup$

                                                                                  Let $ F_0=0, F_1=1 ; F_{n+1}=F_{n-1}+F_n
                                                                                  $ be the Fibonacci numbers



                                                                                  $zeta(s)$ is the zeta function. Then:



                                                                                  $$
                                                                                  prod_{n=1}^{infty}left[(-1)^{n+1}phi F_n+(-1)^nF_{n+1}right]^{n^{-(s+1)}}=phi^{-zeta(s)}
                                                                                  $$






                                                                                  share|cite|improve this answer











                                                                                  $endgroup$













                                                                                  • $begingroup$
                                                                                    This is Brilliant!!!
                                                                                    $endgroup$
                                                                                    – Von Neumann
                                                                                    May 4 '16 at 9:28
















                                                                                  3












                                                                                  $begingroup$

                                                                                  Let $ F_0=0, F_1=1 ; F_{n+1}=F_{n-1}+F_n
                                                                                  $ be the Fibonacci numbers



                                                                                  $zeta(s)$ is the zeta function. Then:



                                                                                  $$
                                                                                  prod_{n=1}^{infty}left[(-1)^{n+1}phi F_n+(-1)^nF_{n+1}right]^{n^{-(s+1)}}=phi^{-zeta(s)}
                                                                                  $$






                                                                                  share|cite|improve this answer











                                                                                  $endgroup$













                                                                                  • $begingroup$
                                                                                    This is Brilliant!!!
                                                                                    $endgroup$
                                                                                    – Von Neumann
                                                                                    May 4 '16 at 9:28














                                                                                  3












                                                                                  3








                                                                                  3





                                                                                  $begingroup$

                                                                                  Let $ F_0=0, F_1=1 ; F_{n+1}=F_{n-1}+F_n
                                                                                  $ be the Fibonacci numbers



                                                                                  $zeta(s)$ is the zeta function. Then:



                                                                                  $$
                                                                                  prod_{n=1}^{infty}left[(-1)^{n+1}phi F_n+(-1)^nF_{n+1}right]^{n^{-(s+1)}}=phi^{-zeta(s)}
                                                                                  $$






                                                                                  share|cite|improve this answer











                                                                                  $endgroup$



                                                                                  Let $ F_0=0, F_1=1 ; F_{n+1}=F_{n-1}+F_n
                                                                                  $ be the Fibonacci numbers



                                                                                  $zeta(s)$ is the zeta function. Then:



                                                                                  $$
                                                                                  prod_{n=1}^{infty}left[(-1)^{n+1}phi F_n+(-1)^nF_{n+1}right]^{n^{-(s+1)}}=phi^{-zeta(s)}
                                                                                  $$







                                                                                  share|cite|improve this answer














                                                                                  share|cite|improve this answer



                                                                                  share|cite|improve this answer








                                                                                  edited May 2 '16 at 14:32









                                                                                  wythagoras

                                                                                  21.6k444104




                                                                                  21.6k444104










                                                                                  answered Apr 29 '16 at 9:46







                                                                                  user334593



















                                                                                  • $begingroup$
                                                                                    This is Brilliant!!!
                                                                                    $endgroup$
                                                                                    – Von Neumann
                                                                                    May 4 '16 at 9:28


















                                                                                  • $begingroup$
                                                                                    This is Brilliant!!!
                                                                                    $endgroup$
                                                                                    – Von Neumann
                                                                                    May 4 '16 at 9:28
















                                                                                  $begingroup$
                                                                                  This is Brilliant!!!
                                                                                  $endgroup$
                                                                                  – Von Neumann
                                                                                  May 4 '16 at 9:28




                                                                                  $begingroup$
                                                                                  This is Brilliant!!!
                                                                                  $endgroup$
                                                                                  – Von Neumann
                                                                                  May 4 '16 at 9:28











                                                                                  3












                                                                                  $begingroup$

                                                                                  Notice that $frac{2}{1+sqrt5}=frac{1}{phi}$



                                                                                  $$int_0^1frac{2}{(1+sqrt5x)^2}dx=frac{1}{phi}$$






                                                                                  share|cite|improve this answer









                                                                                  $endgroup$


















                                                                                    3












                                                                                    $begingroup$

                                                                                    Notice that $frac{2}{1+sqrt5}=frac{1}{phi}$



                                                                                    $$int_0^1frac{2}{(1+sqrt5x)^2}dx=frac{1}{phi}$$






                                                                                    share|cite|improve this answer









                                                                                    $endgroup$
















                                                                                      3












                                                                                      3








                                                                                      3





                                                                                      $begingroup$

                                                                                      Notice that $frac{2}{1+sqrt5}=frac{1}{phi}$



                                                                                      $$int_0^1frac{2}{(1+sqrt5x)^2}dx=frac{1}{phi}$$






                                                                                      share|cite|improve this answer









                                                                                      $endgroup$



                                                                                      Notice that $frac{2}{1+sqrt5}=frac{1}{phi}$



                                                                                      $$int_0^1frac{2}{(1+sqrt5x)^2}dx=frac{1}{phi}$$







                                                                                      share|cite|improve this answer












                                                                                      share|cite|improve this answer



                                                                                      share|cite|improve this answer










                                                                                      answered May 4 '16 at 20:28







                                                                                      user334593






























                                                                                          3












                                                                                          $begingroup$

                                                                                          $$int_0^1 frac{200sqrt5(1-x^2)-300(1-x)^2}{ left[5sqrt5(1+x)^2-15(1-x^2)+2sqrt5(1-x)^2 right]^2}dx=(2phi+1)(phi+2)$$






                                                                                          share|cite|improve this answer









                                                                                          $endgroup$


















                                                                                            3












                                                                                            $begingroup$

                                                                                            $$int_0^1 frac{200sqrt5(1-x^2)-300(1-x)^2}{ left[5sqrt5(1+x)^2-15(1-x^2)+2sqrt5(1-x)^2 right]^2}dx=(2phi+1)(phi+2)$$






                                                                                            share|cite|improve this answer









                                                                                            $endgroup$
















                                                                                              3












                                                                                              3








                                                                                              3





                                                                                              $begingroup$

                                                                                              $$int_0^1 frac{200sqrt5(1-x^2)-300(1-x)^2}{ left[5sqrt5(1+x)^2-15(1-x^2)+2sqrt5(1-x)^2 right]^2}dx=(2phi+1)(phi+2)$$






                                                                                              share|cite|improve this answer









                                                                                              $endgroup$



                                                                                              $$int_0^1 frac{200sqrt5(1-x^2)-300(1-x)^2}{ left[5sqrt5(1+x)^2-15(1-x^2)+2sqrt5(1-x)^2 right]^2}dx=(2phi+1)(phi+2)$$







                                                                                              share|cite|improve this answer












                                                                                              share|cite|improve this answer



                                                                                              share|cite|improve this answer










                                                                                              answered May 5 '16 at 17:32







                                                                                              user335850






























                                                                                                  3












                                                                                                  $begingroup$

                                                                                                  Here is a collection of the series with reciprocal binomial coefficients.



                                                                                                  $$sum_{n=0}^infty (-1)^n left( begin{matrix} 2n \ n end{matrix} right)^{-1}=frac{4}{5} left(1-frac{sqrt{5}}{5} ln phi right)$$



                                                                                                  $$sum_{n=1}^infty frac{(-1)^n}{n} left( begin{matrix} 2n \ n end{matrix} right)^{-1}=-frac{2sqrt{5}}{5} ln phi$$



                                                                                                  $$sum_{n=1}^infty frac{(-1)^n}{n^2} left( begin{matrix} 2n \ n end{matrix} right)^{-1}=-2 ln^2 phi$$



                                                                                                  $$sum_{n=0}^infty frac{(-1)^n}{2n+1} left( begin{matrix} 2n \ n end{matrix} right)^{-1}=frac{4sqrt{5}}{5} ln phi$$



                                                                                                  $$sum_{n=0}^infty frac{(-1)^n}{n+1} left( begin{matrix} 2n \ n end{matrix} right)^{-1}=frac{8sqrt{5}}{5} ln phi-4 ln^2 phi$$



                                                                                                  $$sum_{n=2}^infty frac{(-1)^n}{n-1} left( begin{matrix} 2n \ n end{matrix} right)^{-1}=frac{3sqrt{5}}{5} ln phi-frac{1}{2}$$



                                                                                                  $$sum_{n=2}^infty frac{(-1)^n}{(n-1)^2} left( begin{matrix} 2n \ n end{matrix} right)^{-1}=1-sqrt{5} ln phi+ ln^2 phi$$



                                                                                                  $$sum_{n=2}^infty frac{(-1)^n}{n^2(n^2-1)} left( begin{matrix} 2n \ n end{matrix} right)^{-1}=4ln^2 phi-frac{sqrt{5}}{2} ln phi-frac{3}{8}$$



                                                                                                  A one with $pi$:



                                                                                                  $$sum_{n=0}^infty left( begin{matrix} 4n \ 2n end{matrix} right)^{-1}=frac{16}{15}+frac{sqrt{3}}{27} pi-frac{2sqrt{5}}{25} ln phi $$



                                                                                                  Source here






                                                                                                  share|cite|improve this answer











                                                                                                  $endgroup$


















                                                                                                    3












                                                                                                    $begingroup$

                                                                                                    Here is a collection of the series with reciprocal binomial coefficients.



                                                                                                    $$sum_{n=0}^infty (-1)^n left( begin{matrix} 2n \ n end{matrix} right)^{-1}=frac{4}{5} left(1-frac{sqrt{5}}{5} ln phi right)$$



                                                                                                    $$sum_{n=1}^infty frac{(-1)^n}{n} left( begin{matrix} 2n \ n end{matrix} right)^{-1}=-frac{2sqrt{5}}{5} ln phi$$



                                                                                                    $$sum_{n=1}^infty frac{(-1)^n}{n^2} left( begin{matrix} 2n \ n end{matrix} right)^{-1}=-2 ln^2 phi$$



                                                                                                    $$sum_{n=0}^infty frac{(-1)^n}{2n+1} left( begin{matrix} 2n \ n end{matrix} right)^{-1}=frac{4sqrt{5}}{5} ln phi$$



                                                                                                    $$sum_{n=0}^infty frac{(-1)^n}{n+1} left( begin{matrix} 2n \ n end{matrix} right)^{-1}=frac{8sqrt{5}}{5} ln phi-4 ln^2 phi$$



                                                                                                    $$sum_{n=2}^infty frac{(-1)^n}{n-1} left( begin{matrix} 2n \ n end{matrix} right)^{-1}=frac{3sqrt{5}}{5} ln phi-frac{1}{2}$$



                                                                                                    $$sum_{n=2}^infty frac{(-1)^n}{(n-1)^2} left( begin{matrix} 2n \ n end{matrix} right)^{-1}=1-sqrt{5} ln phi+ ln^2 phi$$



                                                                                                    $$sum_{n=2}^infty frac{(-1)^n}{n^2(n^2-1)} left( begin{matrix} 2n \ n end{matrix} right)^{-1}=4ln^2 phi-frac{sqrt{5}}{2} ln phi-frac{3}{8}$$



                                                                                                    A one with $pi$:



                                                                                                    $$sum_{n=0}^infty left( begin{matrix} 4n \ 2n end{matrix} right)^{-1}=frac{16}{15}+frac{sqrt{3}}{27} pi-frac{2sqrt{5}}{25} ln phi $$



                                                                                                    Source here






                                                                                                    share|cite|improve this answer











                                                                                                    $endgroup$
















                                                                                                      3












                                                                                                      3








                                                                                                      3





                                                                                                      $begingroup$

                                                                                                      Here is a collection of the series with reciprocal binomial coefficients.



                                                                                                      $$sum_{n=0}^infty (-1)^n left( begin{matrix} 2n \ n end{matrix} right)^{-1}=frac{4}{5} left(1-frac{sqrt{5}}{5} ln phi right)$$



                                                                                                      $$sum_{n=1}^infty frac{(-1)^n}{n} left( begin{matrix} 2n \ n end{matrix} right)^{-1}=-frac{2sqrt{5}}{5} ln phi$$



                                                                                                      $$sum_{n=1}^infty frac{(-1)^n}{n^2} left( begin{matrix} 2n \ n end{matrix} right)^{-1}=-2 ln^2 phi$$



                                                                                                      $$sum_{n=0}^infty frac{(-1)^n}{2n+1} left( begin{matrix} 2n \ n end{matrix} right)^{-1}=frac{4sqrt{5}}{5} ln phi$$



                                                                                                      $$sum_{n=0}^infty frac{(-1)^n}{n+1} left( begin{matrix} 2n \ n end{matrix} right)^{-1}=frac{8sqrt{5}}{5} ln phi-4 ln^2 phi$$



                                                                                                      $$sum_{n=2}^infty frac{(-1)^n}{n-1} left( begin{matrix} 2n \ n end{matrix} right)^{-1}=frac{3sqrt{5}}{5} ln phi-frac{1}{2}$$



                                                                                                      $$sum_{n=2}^infty frac{(-1)^n}{(n-1)^2} left( begin{matrix} 2n \ n end{matrix} right)^{-1}=1-sqrt{5} ln phi+ ln^2 phi$$



                                                                                                      $$sum_{n=2}^infty frac{(-1)^n}{n^2(n^2-1)} left( begin{matrix} 2n \ n end{matrix} right)^{-1}=4ln^2 phi-frac{sqrt{5}}{2} ln phi-frac{3}{8}$$



                                                                                                      A one with $pi$:



                                                                                                      $$sum_{n=0}^infty left( begin{matrix} 4n \ 2n end{matrix} right)^{-1}=frac{16}{15}+frac{sqrt{3}}{27} pi-frac{2sqrt{5}}{25} ln phi $$



                                                                                                      Source here






                                                                                                      share|cite|improve this answer











                                                                                                      $endgroup$



                                                                                                      Here is a collection of the series with reciprocal binomial coefficients.



                                                                                                      $$sum_{n=0}^infty (-1)^n left( begin{matrix} 2n \ n end{matrix} right)^{-1}=frac{4}{5} left(1-frac{sqrt{5}}{5} ln phi right)$$



                                                                                                      $$sum_{n=1}^infty frac{(-1)^n}{n} left( begin{matrix} 2n \ n end{matrix} right)^{-1}=-frac{2sqrt{5}}{5} ln phi$$



                                                                                                      $$sum_{n=1}^infty frac{(-1)^n}{n^2} left( begin{matrix} 2n \ n end{matrix} right)^{-1}=-2 ln^2 phi$$



                                                                                                      $$sum_{n=0}^infty frac{(-1)^n}{2n+1} left( begin{matrix} 2n \ n end{matrix} right)^{-1}=frac{4sqrt{5}}{5} ln phi$$



                                                                                                      $$sum_{n=0}^infty frac{(-1)^n}{n+1} left( begin{matrix} 2n \ n end{matrix} right)^{-1}=frac{8sqrt{5}}{5} ln phi-4 ln^2 phi$$



                                                                                                      $$sum_{n=2}^infty frac{(-1)^n}{n-1} left( begin{matrix} 2n \ n end{matrix} right)^{-1}=frac{3sqrt{5}}{5} ln phi-frac{1}{2}$$



                                                                                                      $$sum_{n=2}^infty frac{(-1)^n}{(n-1)^2} left( begin{matrix} 2n \ n end{matrix} right)^{-1}=1-sqrt{5} ln phi+ ln^2 phi$$



                                                                                                      $$sum_{n=2}^infty frac{(-1)^n}{n^2(n^2-1)} left( begin{matrix} 2n \ n end{matrix} right)^{-1}=4ln^2 phi-frac{sqrt{5}}{2} ln phi-frac{3}{8}$$



                                                                                                      A one with $pi$:



                                                                                                      $$sum_{n=0}^infty left( begin{matrix} 4n \ 2n end{matrix} right)^{-1}=frac{16}{15}+frac{sqrt{3}}{27} pi-frac{2sqrt{5}}{25} ln phi $$



                                                                                                      Source here







                                                                                                      share|cite|improve this answer














                                                                                                      share|cite|improve this answer



                                                                                                      share|cite|improve this answer








                                                                                                      edited Jun 13 '16 at 18:50

























                                                                                                      answered Jun 13 '16 at 18:45









                                                                                                      Yuriy SYuriy S

                                                                                                      15.8k433117




                                                                                                      15.8k433117























                                                                                                          2












                                                                                                          $begingroup$

                                                                                                          Not exactly a series, but might also be of interest:



                                                                                                          $$1-frac{1}{phi}=frac{1}{phi^2}=frac{1}{5} left(1+frac{1}{5} left(1+frac{1}{5} left(1+frac{1}{5} left(1+dots right)^2 right)^2 right)^2 right)^2$$





                                                                                                          $$frac{1}{phi^4}=frac{1}{5} left(1-frac{1}{5} left(1-frac{1}{5} left(1-frac{1}{5} left(1-dots right)^2 right)^2 right)^2 right)^2$$



                                                                                                          $$frac{1}{phi^4}=frac{1}{9} left(1+frac{1}{9} left(1+frac{1}{9} left(1+frac{1}{9} left(1+dots right)^2 right)^2 right)^2 right)^2$$






                                                                                                          share|cite|improve this answer











                                                                                                          $endgroup$


















                                                                                                            2












                                                                                                            $begingroup$

                                                                                                            Not exactly a series, but might also be of interest:



                                                                                                            $$1-frac{1}{phi}=frac{1}{phi^2}=frac{1}{5} left(1+frac{1}{5} left(1+frac{1}{5} left(1+frac{1}{5} left(1+dots right)^2 right)^2 right)^2 right)^2$$





                                                                                                            $$frac{1}{phi^4}=frac{1}{5} left(1-frac{1}{5} left(1-frac{1}{5} left(1-frac{1}{5} left(1-dots right)^2 right)^2 right)^2 right)^2$$



                                                                                                            $$frac{1}{phi^4}=frac{1}{9} left(1+frac{1}{9} left(1+frac{1}{9} left(1+frac{1}{9} left(1+dots right)^2 right)^2 right)^2 right)^2$$






                                                                                                            share|cite|improve this answer











                                                                                                            $endgroup$
















                                                                                                              2












                                                                                                              2








                                                                                                              2





                                                                                                              $begingroup$

                                                                                                              Not exactly a series, but might also be of interest:



                                                                                                              $$1-frac{1}{phi}=frac{1}{phi^2}=frac{1}{5} left(1+frac{1}{5} left(1+frac{1}{5} left(1+frac{1}{5} left(1+dots right)^2 right)^2 right)^2 right)^2$$





                                                                                                              $$frac{1}{phi^4}=frac{1}{5} left(1-frac{1}{5} left(1-frac{1}{5} left(1-frac{1}{5} left(1-dots right)^2 right)^2 right)^2 right)^2$$



                                                                                                              $$frac{1}{phi^4}=frac{1}{9} left(1+frac{1}{9} left(1+frac{1}{9} left(1+frac{1}{9} left(1+dots right)^2 right)^2 right)^2 right)^2$$






                                                                                                              share|cite|improve this answer











                                                                                                              $endgroup$



                                                                                                              Not exactly a series, but might also be of interest:



                                                                                                              $$1-frac{1}{phi}=frac{1}{phi^2}=frac{1}{5} left(1+frac{1}{5} left(1+frac{1}{5} left(1+frac{1}{5} left(1+dots right)^2 right)^2 right)^2 right)^2$$





                                                                                                              $$frac{1}{phi^4}=frac{1}{5} left(1-frac{1}{5} left(1-frac{1}{5} left(1-frac{1}{5} left(1-dots right)^2 right)^2 right)^2 right)^2$$



                                                                                                              $$frac{1}{phi^4}=frac{1}{9} left(1+frac{1}{9} left(1+frac{1}{9} left(1+frac{1}{9} left(1+dots right)^2 right)^2 right)^2 right)^2$$







                                                                                                              share|cite|improve this answer














                                                                                                              share|cite|improve this answer



                                                                                                              share|cite|improve this answer








                                                                                                              edited Apr 10 '16 at 11:28

























                                                                                                              answered Apr 10 '16 at 11:05









                                                                                                              Yuriy SYuriy S

                                                                                                              15.8k433117




                                                                                                              15.8k433117























                                                                                                                  2












                                                                                                                  $begingroup$

                                                                                                                  Here is another one




                                                                                                                  $$int_{-infty}^{+infty}e^{-x^2}cos (2x^2)mathrm dx=sqrt{phi piover 5}$$







                                                                                                                  share|cite|improve this answer









                                                                                                                  $endgroup$













                                                                                                                  • $begingroup$
                                                                                                                    Very cool one!!
                                                                                                                    $endgroup$
                                                                                                                    – Von Neumann
                                                                                                                    Apr 3 '17 at 7:54
















                                                                                                                  2












                                                                                                                  $begingroup$

                                                                                                                  Here is another one




                                                                                                                  $$int_{-infty}^{+infty}e^{-x^2}cos (2x^2)mathrm dx=sqrt{phi piover 5}$$







                                                                                                                  share|cite|improve this answer









                                                                                                                  $endgroup$













                                                                                                                  • $begingroup$
                                                                                                                    Very cool one!!
                                                                                                                    $endgroup$
                                                                                                                    – Von Neumann
                                                                                                                    Apr 3 '17 at 7:54














                                                                                                                  2












                                                                                                                  2








                                                                                                                  2





                                                                                                                  $begingroup$

                                                                                                                  Here is another one




                                                                                                                  $$int_{-infty}^{+infty}e^{-x^2}cos (2x^2)mathrm dx=sqrt{phi piover 5}$$







                                                                                                                  share|cite|improve this answer









                                                                                                                  $endgroup$



                                                                                                                  Here is another one




                                                                                                                  $$int_{-infty}^{+infty}e^{-x^2}cos (2x^2)mathrm dx=sqrt{phi piover 5}$$








                                                                                                                  share|cite|improve this answer












                                                                                                                  share|cite|improve this answer



                                                                                                                  share|cite|improve this answer










                                                                                                                  answered Apr 2 '17 at 20:54









                                                                                                                  gymbvghjkgkjkhgfklgymbvghjkgkjkhgfkl

                                                                                                                  1




                                                                                                                  1












                                                                                                                  • $begingroup$
                                                                                                                    Very cool one!!
                                                                                                                    $endgroup$
                                                                                                                    – Von Neumann
                                                                                                                    Apr 3 '17 at 7:54


















                                                                                                                  • $begingroup$
                                                                                                                    Very cool one!!
                                                                                                                    $endgroup$
                                                                                                                    – Von Neumann
                                                                                                                    Apr 3 '17 at 7:54
















                                                                                                                  $begingroup$
                                                                                                                  Very cool one!!
                                                                                                                  $endgroup$
                                                                                                                  – Von Neumann
                                                                                                                  Apr 3 '17 at 7:54




                                                                                                                  $begingroup$
                                                                                                                  Very cool one!!
                                                                                                                  $endgroup$
                                                                                                                  – Von Neumann
                                                                                                                  Apr 3 '17 at 7:54











                                                                                                                  2












                                                                                                                  $begingroup$

                                                                                                                  We can prove the inequalities



                                                                                                                  $$frac{3}{2}<frac{8}{5}<phi<frac{13}{8}<frac{5}{3}$$



                                                                                                                  with representations



                                                                                                                  $$begin{align}phi&=frac{3}{2}+frac{1}{4}int_0^1 frac{dx}{sqrt{4+x}}\
                                                                                                                  \
                                                                                                                  phi&=frac{8}{5}+frac{1}{5}int_0^1 frac{dx}{sqrt{121+4x}}\
                                                                                                                  \
                                                                                                                  phi&=frac{13}{8}-frac{1}{16}int_0^1 frac{dx}{sqrt{80+x}}\
                                                                                                                  \
                                                                                                                  phi&=frac{5}{3}-frac{1}{3}int_0^1 frac{dx}{sqrt{45+4x}}\
                                                                                                                  end{align}$$






                                                                                                                  share|cite|improve this answer











                                                                                                                  $endgroup$


















                                                                                                                    2












                                                                                                                    $begingroup$

                                                                                                                    We can prove the inequalities



                                                                                                                    $$frac{3}{2}<frac{8}{5}<phi<frac{13}{8}<frac{5}{3}$$



                                                                                                                    with representations



                                                                                                                    $$begin{align}phi&=frac{3}{2}+frac{1}{4}int_0^1 frac{dx}{sqrt{4+x}}\
                                                                                                                    \
                                                                                                                    phi&=frac{8}{5}+frac{1}{5}int_0^1 frac{dx}{sqrt{121+4x}}\
                                                                                                                    \
                                                                                                                    phi&=frac{13}{8}-frac{1}{16}int_0^1 frac{dx}{sqrt{80+x}}\
                                                                                                                    \
                                                                                                                    phi&=frac{5}{3}-frac{1}{3}int_0^1 frac{dx}{sqrt{45+4x}}\
                                                                                                                    end{align}$$






                                                                                                                    share|cite|improve this answer











                                                                                                                    $endgroup$
















                                                                                                                      2












                                                                                                                      2








                                                                                                                      2





                                                                                                                      $begingroup$

                                                                                                                      We can prove the inequalities



                                                                                                                      $$frac{3}{2}<frac{8}{5}<phi<frac{13}{8}<frac{5}{3}$$



                                                                                                                      with representations



                                                                                                                      $$begin{align}phi&=frac{3}{2}+frac{1}{4}int_0^1 frac{dx}{sqrt{4+x}}\
                                                                                                                      \
                                                                                                                      phi&=frac{8}{5}+frac{1}{5}int_0^1 frac{dx}{sqrt{121+4x}}\
                                                                                                                      \
                                                                                                                      phi&=frac{13}{8}-frac{1}{16}int_0^1 frac{dx}{sqrt{80+x}}\
                                                                                                                      \
                                                                                                                      phi&=frac{5}{3}-frac{1}{3}int_0^1 frac{dx}{sqrt{45+4x}}\
                                                                                                                      end{align}$$






                                                                                                                      share|cite|improve this answer











                                                                                                                      $endgroup$



                                                                                                                      We can prove the inequalities



                                                                                                                      $$frac{3}{2}<frac{8}{5}<phi<frac{13}{8}<frac{5}{3}$$



                                                                                                                      with representations



                                                                                                                      $$begin{align}phi&=frac{3}{2}+frac{1}{4}int_0^1 frac{dx}{sqrt{4+x}}\
                                                                                                                      \
                                                                                                                      phi&=frac{8}{5}+frac{1}{5}int_0^1 frac{dx}{sqrt{121+4x}}\
                                                                                                                      \
                                                                                                                      phi&=frac{13}{8}-frac{1}{16}int_0^1 frac{dx}{sqrt{80+x}}\
                                                                                                                      \
                                                                                                                      phi&=frac{5}{3}-frac{1}{3}int_0^1 frac{dx}{sqrt{45+4x}}\
                                                                                                                      end{align}$$







                                                                                                                      share|cite|improve this answer














                                                                                                                      share|cite|improve this answer



                                                                                                                      share|cite|improve this answer








                                                                                                                      edited Jun 8 '17 at 21:48

























                                                                                                                      answered Jun 8 '17 at 21:33









                                                                                                                      Jaume Oliver LafontJaume Oliver Lafont

                                                                                                                      3,09411033




                                                                                                                      3,09411033























                                                                                                                          2












                                                                                                                          $begingroup$

                                                                                                                          enter image description here



                                                                                                                          This is my attempt to find another integral representation for Golden ratio using some special function as shown in the above image !!!!!!!






                                                                                                                          share|cite|improve this answer









                                                                                                                          $endgroup$









                                                                                                                          • 2




                                                                                                                            $begingroup$
                                                                                                                            I already saw it in a past question of yours. It's really cool, can you prove it?
                                                                                                                            $endgroup$
                                                                                                                            – Von Neumann
                                                                                                                            Mar 22 '18 at 6:42






                                                                                                                          • 2




                                                                                                                            $begingroup$
                                                                                                                            Can we see all the proof please ?
                                                                                                                            $endgroup$
                                                                                                                            – Abr001am
                                                                                                                            May 8 '18 at 15:28
















                                                                                                                          2












                                                                                                                          $begingroup$

                                                                                                                          enter image description here



                                                                                                                          This is my attempt to find another integral representation for Golden ratio using some special function as shown in the above image !!!!!!!






                                                                                                                          share|cite|improve this answer









                                                                                                                          $endgroup$









                                                                                                                          • 2




                                                                                                                            $begingroup$
                                                                                                                            I already saw it in a past question of yours. It's really cool, can you prove it?
                                                                                                                            $endgroup$
                                                                                                                            – Von Neumann
                                                                                                                            Mar 22 '18 at 6:42






                                                                                                                          • 2




                                                                                                                            $begingroup$
                                                                                                                            Can we see all the proof please ?
                                                                                                                            $endgroup$
                                                                                                                            – Abr001am
                                                                                                                            May 8 '18 at 15:28














                                                                                                                          2












                                                                                                                          2








                                                                                                                          2





                                                                                                                          $begingroup$

                                                                                                                          enter image description here



                                                                                                                          This is my attempt to find another integral representation for Golden ratio using some special function as shown in the above image !!!!!!!






                                                                                                                          share|cite|improve this answer









                                                                                                                          $endgroup$



                                                                                                                          enter image description here



                                                                                                                          This is my attempt to find another integral representation for Golden ratio using some special function as shown in the above image !!!!!!!







                                                                                                                          share|cite|improve this answer












                                                                                                                          share|cite|improve this answer



                                                                                                                          share|cite|improve this answer










                                                                                                                          answered Mar 21 '18 at 23:19









                                                                                                                          zeraoulia rafikzeraoulia rafik

                                                                                                                          2,40311029




                                                                                                                          2,40311029








                                                                                                                          • 2




                                                                                                                            $begingroup$
                                                                                                                            I already saw it in a past question of yours. It's really cool, can you prove it?
                                                                                                                            $endgroup$
                                                                                                                            – Von Neumann
                                                                                                                            Mar 22 '18 at 6:42






                                                                                                                          • 2




                                                                                                                            $begingroup$
                                                                                                                            Can we see all the proof please ?
                                                                                                                            $endgroup$
                                                                                                                            – Abr001am
                                                                                                                            May 8 '18 at 15:28














                                                                                                                          • 2




                                                                                                                            $begingroup$
                                                                                                                            I already saw it in a past question of yours. It's really cool, can you prove it?
                                                                                                                            $endgroup$
                                                                                                                            – Von Neumann
                                                                                                                            Mar 22 '18 at 6:42






                                                                                                                          • 2




                                                                                                                            $begingroup$
                                                                                                                            Can we see all the proof please ?
                                                                                                                            $endgroup$
                                                                                                                            – Abr001am
                                                                                                                            May 8 '18 at 15:28








                                                                                                                          2




                                                                                                                          2




                                                                                                                          $begingroup$
                                                                                                                          I already saw it in a past question of yours. It's really cool, can you prove it?
                                                                                                                          $endgroup$
                                                                                                                          – Von Neumann
                                                                                                                          Mar 22 '18 at 6:42




                                                                                                                          $begingroup$
                                                                                                                          I already saw it in a past question of yours. It's really cool, can you prove it?
                                                                                                                          $endgroup$
                                                                                                                          – Von Neumann
                                                                                                                          Mar 22 '18 at 6:42




                                                                                                                          2




                                                                                                                          2




                                                                                                                          $begingroup$
                                                                                                                          Can we see all the proof please ?
                                                                                                                          $endgroup$
                                                                                                                          – Abr001am
                                                                                                                          May 8 '18 at 15:28




                                                                                                                          $begingroup$
                                                                                                                          Can we see all the proof please ?
                                                                                                                          $endgroup$
                                                                                                                          – Abr001am
                                                                                                                          May 8 '18 at 15:28










                                                                                                                          1 2
                                                                                                                          next


                                                                                                                          Popular posts from this blog

                                                                                                                          Mario Kart Wii

                                                                                                                          What does “Dominus providebit” mean?

                                                                                                                          Antonio Litta Visconti Arese